Jump to content

Wikipedia:Reference desk/Humanities

From Wikipedia, the free encyclopedia

This is an old revision of this page, as edited by 90.201.110.155 (talk) at 11:17, 23 March 2011 (→‎political violence in middle east vs. geopolitics of oil). The present address (URL) is a permanent link to this revision, which may differ significantly from the current revision.

Welcome to the humanities section
of the Wikipedia reference desk.
Select a section:
Want a faster answer?

Main page: Help searching Wikipedia

   

How can I get my question answered?

  • Select the section of the desk that best fits the general topic of your question (see the navigation column to the right).
  • Post your question to only one section, providing a short header that gives the topic of your question.
  • Type '~~~~' (that is, four tilde characters) at the end – this signs and dates your contribution so we know who wrote what and when.
  • Don't post personal contact information – it will be removed. Any answers will be provided here.
  • Please be as specific as possible, and include all relevant context – the usefulness of answers may depend on the context.
  • Note:
    • We don't answer (and may remove) questions that require medical diagnosis or legal advice.
    • We don't answer requests for opinions, predictions or debate.
    • We don't do your homework for you, though we'll help you past the stuck point.
    • We don't conduct original research or provide a free source of ideas, but we'll help you find information you need.



How do I answer a question?

Main page: Wikipedia:Reference desk/Guidelines

  • The best answers address the question directly, and back up facts with wikilinks and links to sources. Do not edit others' comments and do not give any medical or legal advice.
See also:


March 18

modern satire

so nowadays you see people like matt stone and trey parker being compared to other satirists throughout history as a "contemporary swift" and whatnot.

my question is whether it's a unique phenomenon for a satirist to be compared to his predecessors, or if it's something that is always realized in retrospect. did people call swift the contemporary voltaire? voltaire the contemporary chaucer? chaucer the contemporary aristophanes? or is this a newer phenomenon? Jasonberger (talk) 22:03, 23 March 2011 (UTC)[reply]

focus on the value of a job rather than employment in general

have any politicians or philosophers written anything on employment regarding the actual value of a job in society? i find there is a focus keeping unemployment low but if there are no good jobs out there, what good comes out of people working? telemarketing, door men, greeters, people dressed in costumes outside wings restaurants. while these people are employed, their jobs arguably do nothing constructive and only exist because they push money from one business to the other without actually generating a product or service. any wiki articles on value of work and the positive sides to unemployment (if there are any true benefits)? —Preceding unsigned comment added by 74.58.149.102 (talk) 00:20, 18 March 2011 (UTC)[reply]

There are two quite different concepts you might be asking about:
1) Does the job provide workers with self-esteem ? That classic government "busy-work" jobs like moving a mound of dirt, then moving it back, day after day, might be an example of rather futile jobs. However, even then the exercise, fresh air, and having somebody only pay you when you show up and work can do wonders for morale. Obviously a job where you feel you accomplish something is better, though.
2) Does the job create wealth for the society as a whole ? Here some jobs are clear-cut wealth builders, like agriculture, construction, and manufacturing, as almost everyone considers themself wealthier when they have food, a house, and a car. Services are trickier. Some make you feel wealthier, like having maid service, and a doctor, while others do not, like having telemarketers hounding you. Which goods and services can be exported for cash to improve the balance of trade is another, related issue. StuRat (talk) 01:14, 18 March 2011 (UTC)[reply]
(Edit Conflict) Well, the most obvious good is to the people themselves ('society' is composed of people), because they can, more often than not, live better on a wage or salary than on what (society-funded) Government welfare/benefits may be available, and beyond material advantages usually enjoy enhanced personal self-respect and satisfaction by supporting themselves through providing even optional services to others (though I would exempt telemarketers from the latter point). I disagree with your "without actually generating a product or service" since - some charitable activities aside - no business pays people to do things not of expected eventual financial value to itself.
Secondarily, there is a good to the economy as a whole, because every person employed will (beyond various complicated benefits thresholds depending on locality) be able to make a net contribution to the tax base and thus help to fund their society's costs, rather than being a net drain.
Your deeper query verging towards national level economic theory is beyond my scope, but you might find something of interest at Productive and unproductive labour.
To obviate possible ad hominem arguments, I should disclose that I myself currently am, following redundancy, entirely dependent on benefits. 90.197.66.165 (talk) 01:39, 18 March 2011 (UTC)[reply]
To translate that last sentence for Americans: "He's been laid-off and is on Unemployment Compensation". StuRat (talk) 07:51, 18 March 2011 (UTC)[reply]
You might find the McJob article interesting. Astronaut (talk) 16:06, 18 March 2011 (UTC)[reply]
These jobs are forms of advertising, which is itself a form of persuasion and ought to be a form of rational argument and therefore beneficial. (Forms of advertising which use anti-rational methods, like deception and the bullying variety of hard sell, tend to be outlawed.) So I suggest that most of the criticism of advertising lacks merit, and it does good by encouraging discussion of the value of one product over another. You mention "there is focus keeping unemployment low", though, which puzzles me. Whose focus? (The perils of the passive tense!) Usually this kind of phrase refers to government policy, and usually governments don't employ people to dress up as chickens, so something about your question doesn't seem to add up. 81.131.16.236 (talk) 21:14, 18 March 2011 (UTC)[reply]
They might, indirectly, say by lowering minimum wage so that KFC can hire more dancing chickens. As for deception in advertising, that seems to be completely legal in the US, unless the deception rises to the level of an outright lie. Even then, they may still make more money from the lie than the minimal fine they get, if any. Have you seen the pictures of fast food in advertising ? It bears little resemblance to the actual "food". Advertising rarely tries to convey facts, but typically tries to confuse people into buying a product that it isn't in their interest to buy (if it was, ads wouldn't be needed). The only thing good I can see in ads is that some of them are funny. StuRat (talk) 23:03, 18 March 2011 (UTC)[reply]
Camus' Myth of Sisyphus is tangentially related to what the OP wants. Sisyphus was condemned to pushing a rock up a hill for eternity - a pretty meaningless activity - but Camus concludes "one must imagine Sisyphus happy". 129.67.186.200 (talk) 19:16, 19 March 2011 (UTC)[reply]
If someone is willing to pay you to do something, it must be worth something to them at least. And if it's worth something to them, it is almost certainly creating value for them.124.148.51.38 (talk) 00:58, 20 March 2011 (UTC)[reply]

Why don't Arab leaders like Gaddafi ?

The Arab League apparently voted to ask the UN for a no-fly zone over Libya: [1]. I would have expected them to want to protect national sovereignty above all else, since they may be in a similar situation if revolution spreads in their direction. So, why did they vote that way ? StuRat (talk) 01:25, 18 March 2011 (UTC)[reply]

This article says that they made the move because they fear their people's reactions if they don't appear to stand for the people and against tyranny. Marco polo (talk) 01:46, 18 March 2011 (UTC)[reply]
My personal sense is that other Arab leaders kinda tolerate Gaddafi for various politically and economically expedient reasons, but that they also know what others know; that he's batshit insane and that if things went bad, the more rational and clear thinking Arab leaders wouldn't hesitate to throw him under the bus if necessary. In terms of the dictator continuum, hes definately on the Kim Jong Il side of it, if you catch my drift. His relations with Egypt, for example, have always been quite tense, even flaring into open warfare for a short time in the 1970's. His Islamic Legion was a HUGE destablizing effect on his neighbors, the Sudan and Chad, and I'm not sure other arab leaders are terribly proud of what's gone on in those countries under his influence. The entire Darfour mess has been greatly exacerbated by the Islamic Legion directly. If you read through his article, his closest allies read like a who's who of the batshit insane dictators of the last 40 years, without regard for religion or location. He had close ties with both Idi Amin and Slobodan Milosevic, and I'm not sure either endeared him to the rest of the Arab world. If you read Muammar_Gaddafi#Ideology, it contains the statement "Gaddafi is known for his extremely erratic statements, and commentators often express doubt whether he is being sarcastic or just incoherent. Over the course of his four-decade rule, he has accumulated a wide variety of eccentric and often contradictory statements." That probably captures him well. As recently as 2009, he's blasted other Arab leaders in the Arab League, see [2] where he insults King Abdullah of Saudi Arabia. He's not been a great support of Pan-Arabism, but instead has been one of Pan-Africanism, see United States of Africa of which he is a big supporter. So, in summation he's a) batshit insane dictator without a coherant political philosophy b) he has historically bad relations with other Arab states, and has been an obvious destablizing effect on his weaker neighbors. There's bound to be a point where the other Arab states have to say "enough is enough." They may have reached it. --Jayron32 03:06, 18 March 2011 (UTC)[reply]
Gaddafi was a great supporter of Pan-Arabism, until he realised that other Arab states would not permit him to have a leading role in any meaningful union. Only then did he switch to advocating African unity. Warofdreams talk 12:18, 18 March 2011 (UTC)[reply]
In other words, it's transparent that Gaddafi is a great supporter of Gaddafi and little else; yet another reason that no one, even those culturally close to him, are coming to his defense in his hour of need. He's spent decades making his bed, and now he's finally having to lie in it. --Jayron32 16:46, 18 March 2011 (UTC)[reply]
It's worth noting that many Arab and Muslim countries supported the 1991 Gulf War, including Syria, Saudi Arabia, Bahrain, and Pakistan. Solidarity only goes so far when you've someone like Gaddafi or Saddam in your midst. --Colapeninsula (talk) 10:37, 18 March 2011 (UTC)[reply]
He also looks like Michael Jackson. Honestly, why would someone like him? He's also a openly sponsor of terrorism, and no, Arab leaders do not endorse that. 80.58.205.34 (talk) 10:41, 18 March 2011 (UTC)[reply]
Let's not forget that he has also been openly dismissive of the Palestinian cause (and has treated Palestinians in Libya very poorly) and that is also seen as a very negative point in other Arab countries. --Xuxl (talk) 15:01, 18 March 2011 (UTC)[reply]
But that was only after the other Arab states spurned him, as Warofdreams and Jayron mentioned (like when he tried to latch onto the Egypt-Syria union). He was greatly concerned with the Palestinians in the 60s when the Libyan monarchy didn't care... Adam Bishop (talk) 20:28, 18 March 2011 (UTC)[reply]


In the late '70s/early '80s everyone thought Ghadaffi was a dangerous lunatic. Then, once he consolidated power and had little to fear (and after Lockerbie died down), no one seemed to worry too much about him. There was a bit of a reassessment in the West; lots of folks thought he was a dependable player who could be useful as an intermediary. Now that his power is threatened, he's a lunatic again.
I'm not saying he's not a bad guy. Just that he's a lot more trouble when his power is uncertain. So if you're going to pose a threat to that power, you'd better make sure you finish the job.
It reminds me a bit of a Hugo-winning story, Time Considered as a Helix of Semi-Precious Stones, where there was a law-enforcement agency that tracked, not criminals' nuisance value per se, but the first derivative of that nuisance value. Established hoods they left alone; they went in after the up-and-comers. Ghadaffi was an established hood for a long time. --Trovatore (talk) 01:44, 19 March 2011 (UTC)[reply]

I believe I have read that traditionally in English law, a married woman was the property of her husband.

There is relevant information at Wife selling (English custom). Ghmyrtle (talk) 07:39, 18 March 2011 (UTC)[reply]
It may not answer your question directly, but still: Before the Married Woman's property act in 1882, a woman's property was passed to her husband when she married [3]. Though this does not mean that the woman herself was the property of her husband in a literal sense, it goes a long way in suggesting it.--DI (talk) 07:42, 18 March 2011 (UTC)[reply]
I recommend reading the article on coverture--DI (talk) 08:39, 18 March 2011 (UTC)[reply]
and see Married Women's Property Act 1870 and Married Women's Property Act 1882.--Wetman (talk) 12:01, 18 March 2011 (UTC)[reply]
75.24.76.175 -- She was not his property in any meaning of the term comparable to chattel slavery or similar, but the woman's legal identity was submerged in the husband's legal identity -- see Coverture. (As for "wife-selling", that was actually an unrecognized and illegal method of divorce.) -- AnonMoos (talk) 06:40, 19 March 2011 (UTC)[reply]

BABY FACE NELSON

Hello - please find below the correspondance I have been having with yoursleves about "Baby Face Nelson" - any comments on this would be greatfully appreciated - thanks


Dear Frances Hornal,

Thank you for your email. I'm glad you enjoyed the article on Baby Face Nelson, and I appreciate your sharing your memories of the stories you were told in your youth.

I'm afraid, though, that the authors of the article don't actually work for the Wikimedia Foundation. Articles on Wikipedia are written by the public; anyone may volunteer, although there are certain policies in places to try to keep articles accurate and unbiased. This particular article is the result of collaboration between dozens of people, who have been working pretty steadily since 2005.

Wikipedia does, though, have a place where volunteers may be able to see if they can find something similar to the stories you were told. It is a web based forum staffed by volunteers, called the Reference Desk <http://en.wikipedia.org/wiki/Wikipedia:Reference_desk>. Though there is no guarantee that they can provide an answer, they are pretty good at research. If you tell them your story and ask them if they can look into it, they may be able to tell you if they can find anything to confirm it.

Yours sincerely, Maggie Dennis


02/17/2011 14:37 - Frances Hornal wrote:

Hello - after reading your interesting article on Baby Face Nelson - I would like to let you know the stories we have always been told as children

My Grandfather (Charles Leith) whose mother was (Jessie Leith nee Robertson) had a sister (Mary Ann Robertson) who came from the small Island of Bressay on the Shetland Islands, Scotland - Mary emigrated to Either America or Canada in the late 1800's - she married a eastern European at some point and she was the mother of Baby Face Nelson - I also believe that Mary and Jessie's mothers maiden name was NELSON and that is the name Mary and her Husband took as having a foreign sounding name in America in the early 1900's was not an advantage - we actually have a photo of young Mary and the likeness to the pictures you see of Baby Face is striking. I just thought you might be interested and wondered if you had ever heard of anything resembling this story before

- thanks

Best Regards

Frances Hornal

-- Wikipedia - http://en.wikipedia.org --- Disclaimer: all mail to this address is answered by volunteers, and responses are not to be considered an official statement of the Wikimedia Foundation. For official correspondence, please contact the Wikimedia Foundation by certified mail at the address listed on http://www.wikimediafoundation.org —Preceding unsigned comment added by 80.45.152.74 (talk) 09:29, 18 March 2011 (UTC)[reply]

Hello Frances. I'm not exactly sure why the letter you received from the Wikimedia Foundation sent you here. This is a reference desk for general knowledge queries. If you have a particular question about Baby Face Nelson which is not covered by the article, feel free to post it here. If the biographical information you gave in your letter can be verified by independent sources, feel free to add it to the article – it sounds like it could be an interesting addition. Best wishes, --Viennese Waltz 10:11, 18 March 2011 (UTC)[reply]
I've refactored your email to remove the annoying ">" characters inserted by your email software and which were interfering with my ability to read your email clearly. I've also removed your contact details. Unfortunately, this page is highly visible across the internet and is a natural target for spam-bots looking for contact details so they can subsequently deluge you with unwanted emails. Astronaut (talk) 15:06, 18 March 2011 (UTC)[reply]
The place to address your question is really Talk:Baby Face Nelson. However, I can imagine people will ask for proof that your great-aunt really was the mother of Baby Face Nelson - preferably proof that has been previously published in what Wikipedia defines as a reliable source. Astronaut (talk) 15:45, 18 March 2011 (UTC)[reply]

Is there a name for this fallacy?

Other questions on the same theme got me thinking about this one. A certain Australian politician, who shall remain nameless, came up with one of the most inventive pieces of reasoning in recent years. Someone wanted to give poorer families four times as much money for child support as rich families, and the said politician opined that they were saying the children of poor families were four times as important as those of rich ones. I admit it may not have been exactly that, but it was definitely along those lines, and involved the judgement that a policy allocating four times as much money for some set of newborns implied an attitude towards the value of the children themselves. Is there a name for it, and are there any other good examples of such farcical thinking? It's been emotional (talk) 09:48, 18 March 2011 (UTC)[reply]

Probably a Hasty generalization. See List of fallacies and browse for yourself and see if there's one you think fits better. Schyler! (one language) 14:12, 18 March 2011 (UTC)[reply]
I think it's an example of affirming the consequent. There is a valid principle that if something is viewed as more important, then it will get more money -- but it is incorrectly being used in reverse. Looie496 (talk) 17:25, 18 March 2011 (UTC)[reply]
Yes, and specifically, he's ignoring that relative need is also a consideration when allocating money. For example, if you allocate more money to repairing bridge A, which is in imminent danger of collapse, than bridge B, which is in excellent shape, that doesn't mean bridge A is more important, just that those repairs are more urgent. StuRat (talk) 18:19, 18 March 2011 (UTC)[reply]
Another fallacy seems to be that he is only considering one source of child support (the government), while the rich kids presumably have other sources, such as their families, making the total amount spent on them more than the poor kids. StuRat (talk) 18:21, 18 March 2011 (UTC)[reply]
I'd simply call it politics. He no doubt lives in and gets voted in by a relatively wealthy demographic, so he is pandering to the greed of those voters. Remember that politicians don't necessarily believe what they say. They say what they think the most important part of the the audience (to them) wants to hear. That's politics. HiLo48 (talk) 20:36, 18 March 2011 (UTC)[reply]

Thanks, yes it's definitely just politics, on a practical level, but as far as naming it theoretically, I think it fits pretty well with affirming the consequent. Of course we are imposing a logical structure on a rather way out populist comment (though I think perhaps too populist even for his followers), but I expected that would be required. It's been emotional (talk) 01:52, 19 March 2011 (UTC)[reply]

Yes, affirming the consequent looks like a good descriptor. I like it especially because it doesn't require the person presenting the argument to necessarily believe it. He could simply be hoping that enough of his electorate believed it. (Now, all I need, as a fellow Australian, is to figure out who that fine, caring politician was.) HiLo48 (talk) 02:07, 19 March 2011 (UTC)[reply]


The fallacy that best describes this reasoning is the fallacy of division. I.e. just because something is composed of constituent parts does mean that all the properties of the parts are also properties of the whole. The attempt is to make a statement on the importance of particular persons based on the groups they are in. The concept of human worth and tax dollars doesn't work that way.
I'll give you a perfect example: A flat tax is immoral, not fair just because it is supposedly equal. A rich person is able to light their cigar with a dollar bill, whereas a poor person needs the dollar bill, and it is a meaningful unit (e.g. a known and measurable percentage of the poor person's monthly rent). The dollar bill simply doesn't have the same value to a rich person and a poor person. Therefore it is immoral to have a flat tax. The property of value doesn't apply equally in the two cases. This is why we have the concept known as equity.Greg Bard (talk) 04:45, 19 March 2011 (UTC)[reply]
You know... adding "therefore" doesn't really prove your argument as well as you seem to think it does. Hyperbole doesn't help either. The underlying point you're making, badly, is that there's a marginal value of money... that human happiness increases much more quickly for those first few dollars than it does for the latter ones. That argument has merit, and parameters, and is open for debate. Another question I ask... is the Australian government "giving money" to rich families, who are presumably paying taxes for that money? I realize complex tax systems do this all the time, but are we to be especially astounded at this version of it? Shadowjams (talk) 12:22, 19 March 2011 (UTC)[reply]
A less-than-perfect example, more of an argument against the fairness of a per capita tax than that of a flat tax. A flat tax doesn't take $1 from the beggar and $1 from Croesus. That is, a flat tax doesn't take equal amounts from people, it takes equal percentages of their income, which already factors in the ability of the rich to afford to have more taken from them. - Nunh-huh 05:07, 19 March 2011 (UTC)[reply]
Yes, I don't like the term "flat tax", when they really mean a "flat rate tax". The first should really mean a set fee everyone pays, while the second clearly means that the percentage tax is the same. A true "flat income tax", say where everybody paid $10,000 in taxes each year, is so absurd that nobody even considers it (you would have millions of people unable to pay, have others driven to starvation and homelessness by it, and the government wouldn't get nearly enough income to operate, either). A "flat rate income tax", on the other hand, could work, especially if you had large personal deductions (which would effectively make it a progressive tax). Note that, if we are talking about taxes other than income, then a true flat tax may be possible, such as a fixed fee to register your car or pick up your garbage. StuRat (talk) 20:02, 19 March 2011 (UTC)[reply]
Actually, a flat rate tax also places a disproportionally high burden on the poor. At first glance, it sounds really fair, since everyone pays the same amount, say 10%, so if a person has a $20,000 a year income, they pay $2,000 and if someone makes $2,000,000 per year, they pay $200,000. Except that expenses don't necessarily scale with income. If someone is making $20,000 per year, and the minimum they can live on and still meet all their obligations for food and rent and utilities; is $19,000, then even the flat-rate tax screws them. They're short $1,000 per year, which is a huge portion of their income. The person making $2,000,000 per year is living no where near the limit of survival, indeed the 10% of their income makes a much smaller impact on their standard of living than does the person making $20,000 per year. The flat rate tax therefore places an undue burden on the poor people, and lets the rich people off comparatively light. Progressive tax schemes place a higher burden, in terms of tax rate, on people who can absorb the higher obligation without substantially reducing their standard of living. In the example I gave, the flat tax rate would cause the rich person to, say, purchase one less new Maseratti that year. That exact same tax rate causes the poor person to be evicted from their home, or to go hungry. That's why it doesn't work. --Jayron32 00:03, 20 March 2011 (UTC)[reply]
Yes, but with a high flat tax rate and a high personal deduction, you can make a flat tax rate, for all practical purposes, into a progressive tax rate. Consider a 50% tax rate with a $20,000 personal deduction:
INCOME   TAXABLE INCOME   TAX     EFFECTIVE RATE
------   --------------  ------   --------------
 $20K          $0         $0.0        0.0%
 $25K          $5K        $2.5K      10.0%
 $30K         $10K        $5.0K      16.7%
 $40K         $20K       %10.0K      25.0%
 $50K         $30K       $15.0K      30.0%
$100K         $80K       $40.0K      40.0%
So, you could have a simple, yet still progressive, tax system. StuRat (talk) 10:43, 20 March 2011 (UTC)[reply]
Ok the point here is that fallacy of division is the more appropriate fallacy, rather than affirming the consequent. I see that I have struck a nerve with the argument on the immorality of a flat tax. The more simple argument for the flat tax is that rich people do not merely owe more money in raw terms (which a flat tax accommodates as observed above), but rather they owe more of a percentage because the society and economy that our tax dollars supports makes it possible for them to ever be rich and stay rich in the first place. They owe more to society than poor people, not the same because they have more. We don't pay a flat insurance rate do we? No, rich people pay more to insure their property because they have more value to insure. So too with preventing a collapse of society by paying our tax dollars. It prevents them from losing everything, and they have more to lose. Greg Bard (talk) 20:58, 20 March 2011 (UTC)[reply]

Prohibition-five?

In Scrubs, why does the Todd become opposed to high-fiving in the hospital? —Preceding unsigned comment added by 83.70.250.255 (talk) 13:46, 18 March 2011 (UTC)[reply]

In the episode My Full Moon, the Todd bans other people from high-fiving because that's his "thing". Is this what you're referring to? Vimescarrot (talk) 16:32, 18 March 2011 (UTC)[reply]
"His thing"? What, like he invented it? ←Baseball Bugs What's up, Doc? carrots08:25, 19 March 2011 (UTC)[reply]
No. That expression means it's something that only he does, among his circle of friends/coworkers. Or, in this case, he would like to be the only one who does it, among that group. StuRat (talk) 05:28, 21 March 2011 (UTC)[reply]
To clarify, giving high-fives is his "thing", not banning others from doing so. StuRat (talk) 19:18, 18 March 2011 (UTC)[reply]
Thanks! —Preceding unsigned comment added by 83.70.250.255 (talk) 21:57, 18 March 2011 (UTC)[reply]

Ancient language

I know that Sanskrit is Hindus' ancient language and Pali is Buddhists' ancient language, but what about Jains? What is their ancient language? —Preceding unsigned comment added by 70.53.229.65 (talk) 20:55, 18 March 2011 (UTC)[reply]

I thought it was sort of Sanskrit or Sanskrit descendants in general, but check out Jainism#Jain_literature, they wrote in all sorts of languages. And also, Pali is one of the descendants of Sanskrit. Adam Bishop (talk) 22:33, 18 March 2011 (UTC)[reply]

US Health Care, workers paying for elders?

Dear Wikipedia, I have a question.

I've read up on the US health care debate, and am quite perplexed. I noticed one argument that was not fleshed out for me: Donald Barr, seemingly opposed to the status quo of 2000) claimed that [t]he money paid in payroll taxes into the Medicare Trust Fund is not put aside to pay for the care of current workers when they retire. Instead it is used to pay for care of people already retired. - and that this was problematic. Ie, it was implied that the US system (only) did this; that it would somehow be different elsewhere. Don't all European countries do this? As far as I understand, only your pension (Social Security in the US) is 'paid' by you, the individual, in that your work is a factor in deciding its eventual size. However, all your (European) taxes otherwise contribute to what is (although not entirely commensurate) the equivalent of your own, universal Medicare. This was the book "Introduction to US Health Care". Am I confusing something here?

I appreciate your help, as always. 80.213.11.105 (talk) 21:03, 18 March 2011 (UTC)[reply]

Note that a practice can be widespread and still be problematic, especially when the problems only appear years later, like smoking. StuRat (talk) 22:14, 18 March 2011 (UTC)[reply]
Are you confusing something here? I see your reply only tangentially related to my question, unless I misunderstood you. I am first and foremost asking whether the assessment that American taxes to medicare are distributed, or the funds used, differently from other (European?) systems, is correct - see my third last sentence in the main body above. 80.213.11.105 (talk) 23:01, 18 March 2011 (UTC)[reply]
Correct me if I'm wrong, but this appeared to be the argument:
1) US Medicare has current workers pay for health care of retirees.
2) This is problematic, economically, in the long term.
3) Therefore, this problematic practice must not be widespread, but limited to the US.
I was disagreeing with point number 3. StuRat (talk) 19:51, 19 March 2011 (UTC)[reply]
Europe does not have a general system and not all European countries have universal health care (Germany, for example, does not have it). 212.169.189.21 (talk) 01:07, 19 March 2011 (UTC)[reply]
Data point: Sweden's system is just like the one described above, i.e. there is no personal fund for your future health care, nor is money put aside for future use by your age group. The taxes paid in one year pays for the health care produced that year, more or less. Sjö (talk) 07:15, 19 March 2011 (UTC)[reply]
Germany [does not have universal health care] is at best misleading. Germany has a system of public health insurance open to all. Yes, people above a certain income level can opt out, and self-employed people have to explicitly opt in (if they don't buy coverage elsewhere), but overall the system is pretty universal. And if you legitimately cannot afford health care, you still get treated at the same level of care at the public's expense. --Stephan Schulz (talk) 07:24, 19 March 2011 (UTC)[reply]
I know of no government in the world that puts aside funds for future health care (though individuals and insurance companies do so). There might be the odd exception somewhere, but almost all governments fund current care from current taxes etc. Perhaps someone knows if there are exceptions? Dbfirs 07:38, 19 March 2011 (UTC)[reply]
Hong Kong does. The government holds sufficient reserves to pay for healthcare for everyone many times over. The total is about 3.5 time annual payment for all government services, not just healthcare.DOR (HK) (talk) 05:13, 21 March 2011 (UTC)[reply]
Yes, the problem with such a system is that there would be an immediate cost, yet no immediate benefits, and thus the unmet need for health care (which presumably would have driven the creation of the system), would remain unmet, possibly for decades. That's not politically viable. Perhaps a system which gradually transitions from the type we currently have to the more sustainable type might work. StuRat (talk) 22:17, 19 March 2011 (UTC)[reply]
"don't all European countries do this?" was a horrible simplification for me to make, making it more likely that the distribution of money be conflated with the general health care policies.
That aside, I think I should read between Barr's lines: Maybe he is saying that the current policies are/were unsatisfactory because of this economical rule as it were. Ie, one should never depend on the workers's tax output to adjust depending on the ratio of workers and elderly to support. Now, that enables me to ask a question: How, in general, do the European welfare states accomplish this? Are they simply good at keeping their medical costs down, meaning the reduced amount of workers paying £££ and the increased amount of elderly people is a problem, but not as big a problem? Or is the movement of funding drastically different, with perhaps taxes amounting to a smaller part of their budgets?
I think I will ask a new question on US health care below, after searching through the archives for a bit. 80.213.11.105 (talk) 11:23, 20 March 2011 (UTC)[reply]
I believe the nations most successful at containing costs implement price controls. That is, they set the prices that can be charged for medical equipment and procedures, set the pay rates for medical professionals, etc. This means that everyone in the medical field is paid less. The US, on the other hand, relies on competition to keep prices down, which doesn't work, since very few people needing urgent medical care are in a position to shop around for the best price. (Competition does work fairly well for optional medical procedures, provided that all the relevant information is made available to the consumer to enable wise choices.) Once hospitals can no longer pass on their costs, they find ways to save money (hopefully not by cutting anything necessary). Another way to limit medical costs is by rationing, essentially refusing to pay for expensive procedures with little hope of success. Then there's elimination of for-profit companies, such as insurance companies, who all take a "piece of the pie". StuRat (talk) 05:40, 21 March 2011 (UTC)[reply]
It is worth mentioning that the US spends the most on healthcare but doesn't get the best care for all that money (see: this Reuters article). The table on Health care system#Cross-country comparisons make for interesting reading. Astronaut (talk) 20:15, 20 March 2011 (UTC)[reply]

Finance Question about Bond Prices and Compounded Interest Rates

Question moved to "Finance Question about Bond Prices and Compounded Interest Rates)" on the Mathematics desk. -- OP, 68.75.28.230 (talk) 22:13, 18 March 2011 (UTC) [reply]

Source of national debt

On the news I always hear about countries and the massive amounts of debt that they have. The US in in debt, the UK is in debt, Ireland, Greece and so. If everyone is in debt, who are they in debt to? Postrock1 (talk) 22:41, 18 March 2011 (UTC)[reply]

China and corporate and private bondholders, mainly. StuRat (talk) 22:43, 18 March 2011 (UTC)[reply]
Almost all countries owe other countries money, and almost all countries are owed money from other countries. Other countries have made a debt against themselves. Still others owe debts to organizations that they are a member of, like the United Nations or the African Union. Albacore (talk) 23:04, 18 March 2011 (UTC)[reply]
Many of the holders of U.S. governent debt are American individuals and institutions. If you own a savings bond, you own a part of the national debt. -- Mwalcoff (talk) 23:07, 18 March 2011 (UTC)[reply]
Private individuals mostly. Shadowjams (talk) 08:07, 19 March 2011 (UTC)[reply]
To use a specific example, this article offers a rough breakdown of the holders of UK government bonds. In order: insurance and pension funds, then overseas investors, then British banks, then other "financial institutions", then building societies and private investors right down at the bottom.
"Overseas" isn't defined, but looking at the equivalent data for the US, just over a quarter is held by China, 20% by Japan, 6% by the UK, 4% by Brazil, and a long tail falling off after that. Note that this doesn't automatically mean those governments hold the debt, just that someone in that country does; in some cases it might be private financial institutions, in others sovereign wealth funds. Shimgray | talk | 12:32, 19 March 2011 (UTC)[reply]


March 19

Being approached by a child street prostitute

What should one do after the fact? Is there any child protection agency, not the New York City police, one can contact? Thanks. 66.108.223.179 (talk) 02:32, 19 March 2011 (UTC)[reply]

DCFS possibly. If they aren't the "correct" people to call, then they could direct you to who are. Dismas|(talk) 03:41, 19 March 2011 (UTC)[reply]
Contact the police directly - Child Protective Services don't really have the skillset or authority to deal with pimps or criminal activity. they usually step in at the request of police or judges after it's determined that a child needs to be monitored or removed in a given situation. There's no reason not to contact the police (a minor's record will be sealed and/or expunged, so nothing from this will follow him/her into adulthood), unless you think the police are connected to the crime or unwilling to investigate. in that case you should contact a non-profit child welfare advocacy organization (there are dozens of those) and seek out their advice.--Ludwigs2 22:29, 19 March 2011 (UTC)[reply]
"Unless you think the police are connected to the crime"????? In a developed country? You are joking Ludwigs, please, please tell me you are joking. Do you not have cross-agency working in the USA? Surely someone can quickly report this to whatever agency is closest to hand and be absolutely 100% sure that it will be pursued? Itsmejudith (talk) 00:05, 20 March 2011 (UTC)[reply]
There has been police corruption in some U.S. cities, such as cops taking bribes to ignore certain criminal activity. I would hope that no cop would be so cold-hearted as to let child prostitution slide. -- Mwalcoff (talk) 00:42, 20 March 2011 (UTC)[reply]
I'm still alarmed in that you phrase this in terms of the action of one individual cop. There should be systems in place so that a cop would know that it was imperative not to ignore such a thing. In the UK there has been huge public outrage about child abuse, particularly about social workers not doing their jobs properly, but also cases where the police, doctors and schools have failed to notice abuse. We do have some police corruption; there are currently allegations of spying on behalf of the tabloid newspapers. But if there was the slightest suspicion it could affect child protection, the public storm would be massive. Itsmejudith (talk) 11:58, 20 March 2011 (UTC)[reply]
Judith: I wasn't really making that implication overtly, but rather trying to address the OP's reticence to contact police, in a scattershot manner. The only reasons not to want to do that are because you don't want the child involved in a police investigation or you don't trust the police. Most cops in the US are moral sticklers about law and order (sometimes to an unpleasantly aggressive extent), but there are plenty of jaded cops - particularly in urban areas - for whom one more child prostitute is a disagreeable-but-insignificant reality of life, and a few bad eggs who are not above turning a blind eye for the right price. The real problem in the US is that police are extraordinarily cliquish and tend to protect even bad cops from public censure. One has to be fairly determined about being evil to get one's fellow cops to take notice. --Ludwigs2 14:10, 20 March 2011 (UTC)[reply]
"one more child prostitute is a disagreeable-but-insignificant reality of life". Any other rich country where that could be said? By way of comparison, London is gearing up to the Olympics and people are starting to warn that there will be a massive influx of prostitutes. Police are being asked to get the problem of human trafficking under control. We know that, for example, young women in eastern Europe are tricked into coming to Britain on the promise of jobs, and then virtually enslaved. But no-one is saying that under-18s will be significantly involved. The risk of getting caught is not worth it. Itsmejudith (talk) 17:53, 20 March 2011 (UTC)[reply]
That has to do with the peculiar way class distinctions evolved in the US. It's an irony that the US - because it lacks the overt, structured class distinctions that exist in Europe - is utterly blind to implicit class distinctions determined by wealth and poverty, and is much more callous about them. The child of a wealthy family being approached by a stranger is a terrifying outrage that calls for an immediate and strong police response; the runaway child of a poor family ending up as a prostitute is just one of those things that happens to those kinds of people. As an American, I often find my fellow country-people stupid, bigoted, and arrogant; unfortunately, I haven't got the faintest idea what to do about it, and I refuse to resort to the spluttering incomprehensible grunting and growling that passes for political dialog here. fun as that might be... --Ludwigs2 18:24, 20 March 2011 (UTC)[reply]
Child prostitution and sexual abuse happens everywhere, along with adult prostitution. It just tends to be better hidden. And cops will ignore it, for various reasons, as in the following case, where drug users were involved. My brother witnessed repeated child prostitution of very obviously underage children near his work in Cardiff, Wales, about 10 years ago, and kept reporting it, to no effect. Finally, he managed to get hold of a higher ranking officer, who promised to deal with, and it was then sorted, or at least it was no longer happening right by his office. I also know social workers who have commented on its increase, mainly due to children getting addicted to harder drugs. This is still relatively uncommon, but it definitely goes on. I think sometimes people are so shocked, they like to pretend it doesn't happen. Snorgle (talk) 14:57, 24 March 2011 (UTC)[reply]

A rather vague question

I remember reading (somewhere) about an inscription that was discovered upon renovation of a large medieval cathedral. I believe it was on the spire or roof. I can't remember what it said, nor where this discovery occurred. I would be very happy if someone remembers what I've forgotten, that is, what was written, where it was, or both. Thank you. Vidtharr (talk) 03:34, 19 March 2011 (UTC)[reply]

It would help if you told us when you heard this. Was it recent, or years ago?AerobicFox (talk) 05:01, 19 March 2011 (UTC)[reply]
I read it within the last two years. As to the subject matter, I believe it had something to do with the construction of the cathedral and how the laborers would never see it completed. Hopefully this helps narrow things down. Thank you. Vidtharr (talk) 05:08, 19 March 2011 (UTC)[reply]
Well, medieval cathedrals sometimes took centuries to build - for example, Wells Cathedral took over 300 years to build. A more modern example is Sagrada Família in Barcelona which has been under construction since 1882. It is not surprising therefore that those involved in the early construction phases would often never see the building completed. Judging from a suitable Google search, it seems inscriptions are found during many renovations. Without additional details, it might be very hard to track down exactly the one you are thinking of. Astronaut (talk) 10:52, 19 March 2011 (UTC)[reply]
The cathedral that underwent the renovation is in (western) Europe. Google still doesn't seem to be offering any real assistance in the matter. Thank you again. Vidtharr (talk) 17:14, 19 March 2011 (UTC)[reply]

My awesome boyfriend pointed me towards this.http://www.salisburycathedral.org.uk/news.php?id=463..Hotclaws (talk) 11:00, 21 March 2011 (UTC)[reply]

Economics of fundamentalist polygamy

How does a community like Bountiful, British Columbia survive economically? Winston Blackmore has 25 wives and 100 children. I don't think fundamentalist Mormon religion gives the women much opportunity to work. It sounds like a farm designed to produce the most unwanted waste product on the planet - humans. From the documentary I'm watching, they have modern buildings, kitchens, vehicles, own large regions of land. Where does all the money come from? Wnt (talk) 05:24, 19 March 2011 (UTC)[reply]

You consider yourself to be a "waste product"??? ←Baseball Bugs What's up, Doc? carrots06:36, 19 March 2011 (UTC)[reply]
Is there any country in the world that wants people, as opposed to cash (or occasionally, cheap short-term competition to drive down wages in skilled occupations)? Wnt (talk) 08:20, 19 March 2011 (UTC)[reply]
That's about as cynical a viewpoint as I've seen, and it doesn't answer my question anyway: Do you consider yourself to be "waste"? ←Baseball Bugs What's up, Doc? carrots08:23, 19 March 2011 (UTC)[reply]
No idea about British Columbia, but in the 1960s and 1970s some groups in certain U.S. states notoriously manipulated the welfare system. As for a subculture of thriving agricultural population growth, see Hutterites, Old_Order_Amish#Population_and_distribution, etc. AnonMoos (talk) 06:35, 19 March 2011 (UTC)[reply]

I am by no means an expert on The Latter Day Saints, but I think that some sects believe they must create their own paradise become The Return of Christ. Having many children and a self-sufficient community would be fulfilling the commands at Genesis 1:28 and Isaiah 65:17-25, respectively. The answer to your question is two-fold. First thing that comes to mind is the promise of Jesus at Matthew 6:33:

“Keep on, then, seeking first the kingdom and his righteousness, and all these [other] things will be added to YOU."

— Jesus, Book of Matthew

Additionally, this particular British sect of The Latter Day Saints may adhere to some form of Primitive communism. What is the name of the documentary? It sounds interesting. Schyler! (one language) 13:12, 19 March 2011 (UTC)[reply]

They're not British, they're Canadian (although they are originally American). Adam Bishop (talk) 14:53, 19 March 2011 (UTC)[reply]
A big mistake on my part. I just read British... does Wikipedia have an article about the group in question and/or the documentary? Schyler! (one language) 16:55, 19 March 2011 (UTC)[reply]
Yeah, Wnt linked to them in the question (Bountiful is the name of their settlement). Adam Bishop (talk) 17:38, 19 March 2011 (UTC)[reply]
While women may be discouraged from working outside the home, there's plenty of opportunity to make money while working at home. Farming, weaving, crafts, art, etc., could all be forms of income, although I assume that the husband would go into town to sell them. StuRat (talk) 19:42, 19 March 2011 (UTC)[reply]
Raising children is roughly as much work as a full-time job -- but you don't get weekends off. --Carnildo (talk) 00:20, 23 March 2011 (UTC)[reply]

Early human migrations from India

I don't know what exactly Michael Wood means when he says (in The Story of India)that "all non-Africans on the planet can trace their descent from those early migrations into India. The rest of the world was populated from here -- 'Mother India' indeed.". Early human migrations does not present such a simplistic view. What is the scholarly opinion about this statement. 14.139.128.14 (talk) 06:30, 19 March 2011 (UTC)[reply]

While not strictly true that all non-African humans descend from the earliest settlers of the Subcontinent, India does have a role in the early pre-history, see Indus Valley Civilization, which was one of the earliest civilized areas of the world, on par with the Fertile Crescent in terms of being presciently settled and civilized. --Jayron32 12:18, 19 March 2011 (UTC)[reply]
As the map I've added indicates (taken from Recent African origin of modern humans), that idea does not accord with the available data. Looie496 (talk) 17:25, 19 March 2011 (UTC)[reply]
Well, that map is rubbish when it comes to the early Australian population, unless they were awfully good swimmers. HiLo48 (talk) 19:41, 19 March 2011 (UTC)[reply]
I don't think the map is meant to portray exact routes, and being so undetailed it omits many of the islands still existing, let alone areas now submerged by post-glacial sea-level rises, through which the ancestral Australian Aborigines island hopped, presumably using early boats or rafts. {The poster previously known as 87.81.230.195} 90.197.66.165 (talk) 19:57, 19 March 2011 (UTC)[reply]
I noticed that when watching the program, too. Perhaps he was referring to the earlier theory that the "Aryan race" originated in northern India and then spread to Europe  ? Unfortunately, many science/biography/documentary shows on TV seem to lack rigorous fact-checking. StuRat (talk) 19:36, 19 March 2011 (UTC)[reply]

Michael Wood's statement is certainly going to surprise those who settled into the Fertile Crescent, and those who moved from there to Europe or Central Asia. DOR (HK) (talk) 05:21, 21 March 2011 (UTC)[reply]

Japan: America's second greatest ally in the world?

I was listening on the radio yesterday about the disaster in Japan, and this host (I don't remember his name) said that Japan has become America's biggest ally after Great Britain. Is this host right? Willminator (talk) 17:35, 19 March 2011 (UTC)[reply]

Define "biggest". Japan has the second- or third-largest GDP, higher than the UK. They're way down the list of military size. They're pretty high on military spending, but the UK, France, and Germany are all higher. Canada, Brazil, Australia, and others have them beat handily in size. The political aspects don't really have an objective form of measurement, but I don't think there's a clear-cut hierarchy that puts Japan second. — Lomn 18:04, 19 March 2011 (UTC)[reply]
By asking if Japan was our "second greatest ally," I was intending to ask if Japan was our second closest friend (Great Britain currently being the first). Is Japan currently America's second closest friend? If not, where would Japan be today on America's friendship scale? I keep hearing President Obama and others saying that Japan is one of our closest allies, which is something I've heard before the Japanese disaster, and is true. The radio host I heard yesterday said that Japan was America's second greatest (closest) ally. That's the first time I've heard that claim, so I'm wondering if that claim is true. Willminator (talk) 18:50, 19 March 2011 (UTC)[reply]
That's all rather subjective (opinion). The US does, however, have major military bases there, so that's a sign of their usefulness to the US. StuRat (talk) 19:31, 19 March 2011 (UTC)[reply]
So, since it's subjective to rate Japan or some other country as our second closest ally in the world, would it be also subjective to rate Great Britain as our closest ally in the world? Willminator (talk) 19:52, 19 March 2011 (UTC)[reply]
Yes. HiLo48 (talk) 20:13, 19 March 2011 (UTC)[reply]
Yes, and I would argue that Canada is, due to a massive amount of trade between the two, and amazingly few border squabbles for two nations which share one of the longest borders in the world, and miltary co-operation such as NORAD and the DEW line. StuRat (talk) 22:09, 19 March 2011 (UTC)[reply]
Hard to say who is exactly the closest, but the Japanese are definitely in the top 2-3 of the closest allies(in a lot of ways I would consider them the closest). We do have an article Japan – United States relations, but that doesn't outline cultural aspects.AerobicFox (talk) 21:29, 19 March 2011 (UTC)[reply]
I see the OP asked on the talk page there before coming to the ref desk. 213.122.34.134 (talk) 21:37, 19 March 2011 (UTC)[reply]
I already erased my question on the talk page now that I'm getting answers here. No one replied to my question on the talk page, so I decided to erase it when I got answers here. Willminator (talk) 16:41, 21 March 2011 (UTC)[reply]
Oh, please stop now. It's a very silly discussion. There is obviously no precise definition of degree of closeness of alliedness. It's language used by diplomats and politicians, and we know they consciously avoid precise definitions. If we ever really need words here, "A close ally of..." is fine. HiLo48 (talk) 21:38, 19 March 2011 (UTC)[reply]
I sometimes wonder about this sort of thing. Does Wikipedia have a policy on vacuous statements made by reliable sources? I suppose the thing to do would be to write "Politician X said this", with an implied "(whatever that really means)". 213.122.34.134 (talk) 21:45, 19 March 2011 (UTC)[reply]
Well, we could come up with a definition of "closest ally", say the one to which you have the most dollars in trade ? However, as no definition is used universally, there's no way to apply this to establish the "truth" of statements about "closest allies" from politicians. StuRat (talk) 22:05, 19 March 2011 (UTC)[reply]
Australian politicians like to say that America has no stronger friend in the world than Australia, and it would be undiplomatic for US Presidents to disagree. Julia Gillard spoke to the US Congress recently and went on emotionally about how Americans can do anything; I've never heard her or any other Australian politician tell Australians anything remotely like that, so that's quite some praise. Obama has been scheduled to visit Australia ever since he came to office, but at least 2 visits have had to be put off due to other emerging issues, and I'm not aware there's any visit at all scheduled at present. It's in the "We'd love to come Down Under when time permits" basket, which has a commitment value of zero. So, what that all says about the closeness of the US-Australian alliance, I'm really not sure. We have to be content with Oprah and her excessive (even for her) perorations of naked abject lust for all things Australian. -- Jack of Oz [your turn] 00:03, 20 March 2011 (UTC)[reply]
It's not a question of affection, Jack. But you know, Japan is our regional counterweight to China. When New Zealand starts giving us trouble, you can bet Australia will be higher on the returned-call list. --Trovatore (talk) 01:59, 20 March 2011 (UTC)[reply]

What's a no-fly zone?

I read on the BBC that French jets have begun enforcing the no-fly zone in Libya. I don't quite understand why they have "destroyed a number of tanks and armoured vehicles". Were they trying to prevent the tanks from flying, or is the no-fly zone misnamed? 213.122.34.134 (talk) 20:18, 19 March 2011 (UTC)[reply]

I guess we should be referring to No-fly zone, but it doesn't suggest that destroying tanks is part of it. Maybe the media doesn't understand either. HiLo48 (talk) 20:22, 19 March 2011 (UTC)[reply]
(ec)Any hostile action from ground forces towards the planes enforcing the 'no fly zone' (including just locking on radar) would be met with an immediate response. However, from the terms of the UN resolution I believe it also allows any action to prevent harm to the civilian population short of landing ground forces. Mikenorton (talk) 20:25, 19 March 2011 (UTC)[reply]
Those Libyans can be very sneaky... best to play it safe.
OK, seriously, the UN resolution that the international community is now enforcing calls for more than just a "no-fly" zone. It also calls for a halt to ground attacks. The French strikes are designed achieve that. Blueboar (talk) 20:27, 19 March 2011 (UTC)[reply]
Libyan no-fly zone explains, though. WHAAOE! Thanks. 213.122.34.134 (talk) 20:29, 19 March 2011 (UTC)[reply]
Maybe we can be the first to come up with a better name for this style of operation.... HiLo48 (talk) 20:31, 19 March 2011 (UTC)[reply]
Let's call it a ten foot pole operation. 213.122.34.134 (talk) 20:36, 19 March 2011 (UTC)[reply]
Well, really, the old-fashined term for this is balance of power, where in this case the UN is using its forces to counterbalance the extreme equipment advantages the pro-government forces have over their own people. Balance of power has historically been used as a perverse stabilizing influence - making victory questionable enough so that governments hesitate to engage in conflict.
Plus, that way we can use the ultra cool term: it's a BOP-zone! --Ludwigs2 22:21, 19 March 2011 (UTC)[reply]

I think confusing names on Wikipedia are a bit of an issue. If anyone else agrees, especially about this one, maybe continue discussion at Talk:Libyan no-fly zone#Rename. HiLo48 (talk) 21:02, 19 March 2011 (UTC)[reply]

The mention of preventing tanks from flying made me laugh, but it's worth mentioning that sometimes tanks have indeed been known to fly; see Winged tank. --Demiurge1000 (talk) 21:38, 19 March 2011 (UTC)[reply]

Wealth of the Windsor family

If they were no longer paid public money, and had to leave all publicly owned property, how much income and capital would they have left?

From the amount we pour into them, they must realy be on their uppers. 92.24.178.214 (talk) 21:39, 19 March 2011 (UTC)[reply]

Taking this as a serious question, the Duchy of Cornwall made £16 million last year (all those biscuits, presumably). It makes a lot of difference how the "Windsor" family were privatised; the Duchy itself is automatically in the control of the heir of the monarch. I have a feeling, though, this is another of the monarchy-bashing civil list questions we had a while back. It's impossible to answer. Grandiose (me, talk, contribs) 22:32, 19 March 2011 (UTC)[reply]
The "heir of the monarch" being currently Prince Charles, Prince of Wales. I just mention this because it's become flavour of the month in the media to refer to Charles's son William as "heir to the throne". William does not, however, have that distinction. He's in the line of succession, and in the ordinary course of events he will become heir when his father becomes king, and then succeed his father - in about 30 years time. But his father's still waiting patiently in line himself; he might get to become king when he's in his 70s, the way his mother's going. If she lasts as long as her own mother, Charles would not get to become king till about 2026, when he'll be 78. William would then be 44. He might not get to become king himself till he's about 66. -- Jack of Oz [your turn] 23:51, 19 March 2011 (UTC)[reply]
...unless Charles realizes that he's quite a good Prince of Wales and steps aside for a young king, retraining his title and the Duchy...--Wetman (talk) 09:54, 20 March 2011 (UTC)[reply]
If William were to have children, and Charles stepped aside in favor of him, Charles would no longer be the heir to the throne, and therefore no longer eligible to hold the titles of Duke of Cornwall or Prince of Wales. Corvus cornixtalk 18:25, 20 March 2011 (UTC)[reply]
But why would Charles step aside, and why should he? The history of the British monarchy contains no precedent for such a thing (Edward VIII is not a comparable case). The people of the Commonwealth do not want another Richard Nixon. He's been preparing all his life to become king. It's not a popularity contest. The Parliament might just have the teensiest opinion on such a matter, by the way; at law, it's not even Charles's decision. Anyway, it wouldn't depend on whether or not William had progeny. If he were childless, Harry would become heir. -- Jack of Oz [your turn] 19:07, 20 March 2011 (UTC)[reply]
OK, I completely don't get the Nixon ref. Who was the first Nixon the people of the Commonwealth didn't want, or at least don't want another of? Was it the actual Nixon, who I don't see what he has to do with this, or Edward VIII, whose connection with Nixon is obscure to me, or someone else, or some other kind of metaphor altogether? --Trovatore (talk) 08:26, 21 March 2011 (UTC)[reply]
Sorry about that. I was thinking Edward VIII and Richard Nixon are the only people to have resigned voluntarily from their roles as head of state of the UK and the USA. Just as the USA would not welcome another presidential resignation, the UK and the Commonwealth would not welcome another abdication (or whatever it's called when the heir to the throne decides he does not want to succeed and passes the baton to the next in line). I confused matters when I excluded Edward VIII up front (on the grounds that his abdication as king is not the same thing as the resignation of the heir to the throne from the line of succession), but then implicitly included him as an example of an abdication in more general terms. Don't quite know what I was thinking there. Warning: The foregoing may be no clearer than before, in which case, I'm again sorry. -- Jack of Oz [your turn] 11:00, 21 March 2011 (UTC)[reply]
Oh, I see, now that's clear. I didn't make the connection because Nixon's resignation was under shameful circumstances, whereas Edward's abdication was not. --Trovatore (talk) 21:53, 21 March 2011 (UTC)[reply]
But Miesianical (below) reminds me that it's not even possible for an heir to the throne to "de-heir" himself (barring becoming or marrying a Catholic). Well, it's technically possible, but it would require the concurrence of the UK Parliament and the parliaments of all the other 15 Commonwealth realms to make any change to the law concerning the succession to the throne. If Charles became a convicted rapist or a serial killer or bank robber or was declared incurably insane, then maybe they'd consider he'd lost the plot and it would be best if he were removed from the line - but barring such extraordinary circumstances, it just ain't gonna happen. It certainly isn't going to happen just because people generally seem to like William better than Charles. So, all this speculation we keep hearing is just ill-informed gossip-magazine stuff. Whether anyone including Charles likes it or not, Charles will succeed Elizabeth unless he happens to die before her. -- Jack of Oz [your turn] 22:30, 21 March 2011 (UTC)[reply]
I believe that historians 'today' see Edward VIII's actions as shameful. They see him as having been a quitter. GoodDay (talk) 00:45, 22 March 2011 (UTC)[reply]
Well, I don't know what else he should have done. Should he have thumbed his nose at the government's threat to resign, and married Wallis anyway? Or should he have sacrificed his happiness to carry on in the family business, a meaningless job that no one had ever asked him whether he wanted?
Granted, the story's not perfect; it would have been better if he'd made a clean break with royalty and gone and found something to do with his life afterwards. It would also have been better if he hadn't given people reason to suspect he was pro-Nazi. But neither of those things led to his abdication. I can't believe anyone would seriously suggest that that in itself was shameful. --Trovatore (talk) 08:06, 22 March 2011 (UTC)[reply]
There is speculation that Charles might decline to be King if Camilla is not made Queen. Whether that speculation has any basis in fact is another matter. Ultimately the speculation is fun, but pointless... We will find out when Elizabeth II dies. Blueboar (talk) 19:30, 20 March 2011 (UTC)[reply]
That doesn't sound right. If Charles becomes king, Camilla or whoever he's married to at the time (barring a same-sex partnership) WILL BE queen. But HE has made the decision, at least for now, that when he becomes king, she will not be CALLED "Queen" but "Princess Consort". It's entirely in his hands. When the time comes, who knows what will happen. If he has to wait another 15 years, there'll be a whole pile of his subjects who've never even heard of Diana, so the whole point of the alternative title for Camilla will have dissipated. -- Jack of Oz [your turn] 19:48, 20 March 2011 (UTC)[reply]
As Jack says, the wife of a king is a queen. It's part of the definition of the word "queen", really. Where there is some leeway is in what style a particular individual uses. That would be a decision for Charles and Camilla to make (perhaps with the advice of ministers and courtiers). It's the same as the decision for Camilla to be styled "Duchess of Cornwall" rather than "Princess of Wales". She is the Princess of Wales but she doesn't use that title. --Tango (talk) 22:13, 20 March 2011 (UTC)[reply]
In these days of gender equality, is it fair the "the wife of a king is a queen", but " the husband of a queen is a king" doesn't hold? Dbfirs 08:52, 21 March 2011 (UTC)[reply]
But it's also not symmetrical. That is, Queen Elizabeth II's husband, Prince Philip, Duke of Edinburgh, is not considered to be a "king". I suspect that the reason is that, historically, if there was a king and a queen, the king was presumed to have the most power, with the queen in a secondary role. StuRat (talk) 08:49, 21 March 2011 (UTC)[reply]
The Windsor wealth is slap in the face to democracy. They're getting paid for being who they are (royalty). As for the succession? the 16 Commonwealth realm Parliaments decide, not the monarch or anybody within her family. So far, Charlie is still heir-apparent. Legally, Elizabeth II is the heir, as heir means to be in possesion of 'or' in succession to - i.e. Elizabeth II is the 'heir' to George VI. GoodDay (talk) 12:25, 21 March 2011 (UTC)[reply]

"The Windsors" are not paid public money. Those in the Royal Family who carry out official duties on behalf of any of the Commonwealth realms have the expences incurred by those activities paid for by the relevant state, just as it is for any other government official. These funds are controlled by the various parliaments; the monarch hasn't held the purse strings for more than two hundred years. (So much for a "slap in the face" to democracy!) How they earn money as private individuals depends on who it is we're talking about; as has already been noted, Charles has the Duchy of Cornwall, the Queen has the Duchy of Lancaster and investments. But, how much the family is collectively worth, in that sense, I've no idea. Just like any other individual, its their private business.

As for Charles: the question of his "stepping aside" in favour of William is hardly worth pondering; he cannot willingly abdicate his place in the succession. He must have the approval of all the parliaments of the 16 Commonwealth realms (plus each of the 10 provincial legislatures in Canada and perhaps the states of Australia) to do such a thing. It's technically possible, but is such a complex matter he'd likely be dead before it was finally done. --Ħ MIESIANIACAL 13:14, 21 March 2011 (UTC)[reply]

Monarchies got go. They're a hypocracy to democracy. GoodDay (talk) 13:49, 21 March 2011 (UTC)[reply]
Catchy. But wrong, as proven by all the democracies that incorporate monarchy. Plus, history tells us monarchy probably isn't going anywhere; its too flexible to go extinct. --Ħ MIESIANIACAL 13:52, 21 March 2011 (UTC)[reply]
Too archaic, it's gotta go. GoodDay (talk) 14:15, 21 March 2011 (UTC)[reply]
You seem to be under the mistaken belief that democracy is some new invention and ancient and modern are inherently incompatible. --Ħ MIESIANIACAL 16:04, 21 March 2011 (UTC)[reply]
We're in the 21st century, time to stop playing the 'princess in the palace' fairy-tale stuff. GoodDay (talk) 17:01, 21 March 2011 (UTC)[reply]
Perhaps. Not really relevant to the "monarchy is archaic and thus incompatible with democracy and thus must go" argument, though. --Ħ MIESIANIACAL 17:22, 21 March 2011 (UTC)[reply]
British, Australian, Canadian etc, don't get to choose their Head of State. They don't even get to choose their head of government, the Monarch has that choice. GoodDay (talk) 19:11, 21 March 2011 (UTC)[reply]
You seem to be under the mistaken belief that the reference desk is a debate forum. Corvus cornixtalk 19:14, 21 March 2011 (UTC)[reply]
OK, Mies & I will cease. GoodDay (talk) 19:17, 21 March 2011 (UTC)[reply]
The queen inherited billions of pounds of investments including billions of pounds of rent-yielding properties in London. The royal family needs to continue to be exempt to death duties to preserve the value of their investments.
Sleigh (talk) 15:08, 21 March 2011 (UTC)[reply]
Is that really true? They have never paid death duties? That's a disgrace. 92.15.23.133 (talk) 17:34, 22 March 2011 (UTC)[reply]
In 2001 Forbes.com valued her wealth at $420 million but compared to Beatrix of the Netherlands who was supposed to be worth $3.2 billion, Elizabeth needs to get the begging bowl out!. However The Times newspaper in 2005 valued the Royal Family at £6 billion--that's pounds, not dollars mind--if they were to be floated on the stock market. However they can't be so they finally came to the conclusion that their net worth was £300 million. That's not a million miles from Forbes estimate. -Bill Reid | (talk) 19:48, 21 March 2011 (UTC)[reply]
...and that means that Bill Gates could buy them out 100 times, and he does not have several rustic tourists attractions to maintain. Puts things into perspective... --Stephan Schulz (talk) 22:37, 21 March 2011 (UTC)[reply]
Mr. Gates worked hard and deserved some at least of his money by making computers available to almost everyone. What have the Windsors done, apart from continuous PR displays to preserve their own positions? 92.15.6.157 (talk) 10:54, 22 March 2011 (UTC)[reply]
I wasn't aware that Gates was primarily responsible for "making computers available to almost everyone." What makes you think that? --Demiurge1000 (talk) 20:28, 22 March 2011 (UTC)[reply]

Outlook for the USD

What is the outlook for the USD against the other major currencies over the next few months? Thanks. Leptictidium (mt) 23:26, 19 March 2011 (UTC)[reply]

Probably not too bad, but you never know. Itsmejudith (talk) 00:00, 20 March 2011 (UTC)[reply]
I don't mean to be flippant, but with currencies, bond yields, stocks, whatever, you're usually going to have about as many people thinking they should go up as think they should go down. If it were obvious the dollar should be higher, it would already be there. -- Mwalcoff (talk) 00:55, 20 March 2011 (UTC)[reply]
Over the next few months the yen will be falling, so the USD will be doing better compared to that. Apart from that it is faring as usual.AerobicFox (talk) 05:30, 20 March 2011 (UTC)[reply]
Nobody knows. See Efficient markets hypothesis. If they think they know, they are either extremely clever and as a result very rich or, and far more likely, gamblers or charlatans selling financial snake-oil.
On second thoughts I think the best forecast for the USD is its current value, and weighted into the future by the comparative interest rates of the currencies it is compared with. This is what the EMH implies. There must be an article about that but I cannot think what it would be called. Edit: Interest rate parity.
If the OP is thinking about whether to buy foriegn currency now or later, then buying half now and half when needed would reduce the psychological regret. 92.28.241.202 (talk) 13:52, 20 March 2011 (UTC)[reply]

@AerobicFox, the Yen won't necessarily fall. Japaneses are bringing money home from abroad. That means selling Euros and Dollar and buying Yens. Quest09 (talk) 18:02, 20 March 2011 (UTC)[reply]

That has already been taken account of in the current value of the yen. In other words its too late to anticipate what Quest09 has described. 92.28.241.202 (talk) 19:43, 20 March 2011 (UTC)[reply]
Not all price movements are speculative. Sometimes people buy stuff because they need it. Speculators would never ever be able to know in advance how much more Yens will be flowing to Japan in the next months. They are just what they are: speculators trying to guess. Of course, economists will provide you a theory about why everything is like a clock in economics...212.169.189.144 (talk) 01:47, 21 March 2011 (UTC)[reply]
Future known movements are already reflected in current prices due to Temporal arbitrage temporal Arbitrage. 92.15.25.108 (talk) 13:29, 21 March 2011 (UTC)[reply]

Since we are very careful not to offer legal advice, and react in horror at the thought of using these pages to offer medical advice, why oh why do people persist in offering financial advice? Please, people! Leave it to the professionals, and discourage this sort of question! DOR (HK) (talk) 05:26, 21 March 2011 (UTC)[reply]

...or the astronomers, who sometimes are equally skilled to predict market developments as economists... --Soman (talk) 18:35, 22 March 2011 (UTC)[reply]
Or monkeys, dice, or flipping coins. The latter in particular have an impressive 50% success rate, and portfolios chosen by monkeys often beat those of experts. See for example http://www.dailymail.co.uk/news/worldnews/article-1242575/Lusha-monkey-outperforms-94-Russia-bankers-investment-portfolio.html and others. 92.15.14.45 (talk) 10:28, 23 March 2011 (UTC)[reply]


March 20

Were Hitler and Stalin ever photographed together?

See topic. Thanks. The Masked Booby (talk) 01:25, 20 March 2011 (UTC)[reply]

No. They never met officially. There are whacked out theories that they may have secretly walked past each other at one point or another, but it would be a huge stretch of reality to assume someone was standing around with a camera waiting to see if they might walk down the hall at the same time. -- kainaw 01:30, 20 March 2011 (UTC)[reply]
Hitler and Stalin never met face to face (and so would not have been photographed together). All contact between Russia and Germany was handled by their respective foreign ministers.... or by their armies.
That said, there are photographic images that depict them together... all obvious fakes. Blueboar (talk) 01:37, 20 March 2011 (UTC)[reply]
There must be photographers in the 7th Circle of Hell. Clarityfiend (talk) 02:16, 20 March 2011 (UTC)[reply]

Why Libya?

Do any of our Wikipedia articles discuss what exactly motivated the US, Britain, and France to attack Libya? Many governments around the world currently sanction human rights violations like those committed in Libya (countries that come to mind include Sudan, Bahrain, and Yemen), so is it safe to assume that economic or political motivations prompted this attack? If so, what are they? Edge3 (talk) 03:05, 20 March 2011 (UTC)[reply]

I'm not an expert, but I think that the US, UK, and France were just the best equipped and best prepared nations to take military action. BurtAlert (talk) 03:10, 20 March 2011 (UTC)[reply]
I think the questioner is asking why attack Libya and not Sudan, Bahrain, or Yemen, which have documented human rights violations. Wikipedia's best evidence are the infoboxes at the top right of the articles. Sudan and Yemen are both republics and Bahrain is a Kingdom whereas Libya was an autocracy (now it is "disputed," according to the infobox). Schyler (one language) 03:24, 20 March 2011 (UTC)[reply]
"Autocracy" is not incompatible with republics or monarchies. North Korea is an autocratic republic, for example. It's a bit hard to determine how to classify Libya because some people would consider a head of state that is at least ostensibly elected to be a requirement. However, it's not really clear who the official head of state in Libya is under Gadhafi's regime. Gadhafi himself has never been elected to anything. As far as the original question goes, the situation in Libya right now is more serious than the situations in Bahrain and Yemen, with thousands of lives at stake. And Gadhafi's regime has long been detested by both the West and by other Arab countries, making it easier to get international support for military intervention. ASudan is a little different story from Libya because it is an ethnic conflict rather than a widespread uprising against the regime. There is a United Nations/African Union peacekeeping force in Darfur. -- Mwalcoff (talk) 03:48, 20 March 2011 (UTC)[reply]
Your cynicism is healthy, OP, but it gets much more complicated. And, it's like asking, "Since the right thing doesn't happen all the time, does that mean the right thing can never happen...?" ;) WikiDao 03:31, 20 March 2011 (UTC)[reply]

A similar but related question is - Why now, and not ten years ago? HiLo48 (talk) 03:37, 20 March 2011 (UTC)[reply]

Ten years ago the Libyan people weren't desperately requesting it. It would have been a "foreign invasion" then -- and there are only so many "Iraqs" and "Afghanistans" that the world (or even just the US alone) can deal with at one time... WikiDao 03:46, 20 March 2011 (UTC)[reply]
The right autocrats (ones friendly to the west) are already in charge in Yemen and Bahrain, at least. There's no need to replace them. Adam Bishop (talk) 09:44, 20 March 2011 (UTC)[reply]
In diplomacy you have to play the cards you're dealt.You can't ask for a more playable hand or a better partner.--Wetman (talk) 09:50, 20 March 2011 (UTC)[reply]
You can, however, refuse to play in most situations. --Stephan Schulz (talk) 10:11, 20 March 2011 (UTC)[reply]
The pivotal moment that led to the attacks was when the Arab League voted for it: [4]. As for their motivations, I asked the same question a couple days ago: [5]. StuRat (talk) 10:49, 20 March 2011 (UTC)[reply]
I've just read this: http://www.bbc.co.uk/news/world-us-canada-12792637 92.28.241.202 (talk) 14:00, 20 March 2011 (UTC)[reply]
There's a political dimension to this. Libya is a technically socialist state that has been implicated in terrorist attacks (e.g. the Lockerbie plane bombing), and has been overtly hostile to the western states since the fall of the old kingdom. Bahrain, Yemen, Sudan, Saudi Arabia and etc. are either kingdoms or puppet republics that have had a much more pro-west attitude (mostly because they know which side of their bread is buttered). We in the west tolerate autocrats much better than we tolerate socialists, and have historically groomed these tinpot dictatorships to try to maintain a certain stability in the region so that oil production is not affected.
Blame it on cold war politics, which generally held that it was better to have a controllable devil wrapped in liberal camouflage in places like this, than to actually construct a liberal society and risk that the people might choose socialist or religious fundamentalist governments. --Ludwigs2 14:26, 20 March 2011 (UTC)[reply]
Britain is a socialist state too. 92.28.241.202 (talk) 15:24, 20 March 2011 (UTC)[reply]
Read socialism. No, it's not, unless the means of production are publicly controlled. I also doubt that Libya is a "socialist" state in any meaningful name. However, socialism has become a catch phrase for "states unfriendly to the US and not completely laissez-faire", I guess. --Stephan Schulz (talk) 16:02, 20 March 2011 (UTC)[reply]
A lot of the "means of production" are publically controlled, particularly when you consider that Britain is a service economy rather than a metal-basher. For example our "socialised medicine", and that a large proportion of people work for the government.92.28.241.202 (talk) 19:50, 20 March 2011 (UTC)[reply]
Countries can't be re-identified as "socialist" (or any other term) just on the basis of woolly generalisations like this. There are indeed parts of the UK (e.g. Liverpool) where a surprising percentage of the population are either employed by the government or primarily dependent on state hand-outs. However, that doesn't change the fact that the vast majority of the economy (as a whole) is not government controlled. For it to be a socialist country, it would have to be the opposite way round. --Demiurge1000 (talk) 20:20, 20 March 2011 (UTC)[reply]
It wasnt so long ago that one of the two main parties used to sing the "Red Flag" at party conferences. 92.28.250.93 (talk) 22:29, 21 March 2011 (UTC)[reply]
Indeed, in recognition of the hopes of some of their party members that the party would somehow transform a non-socialist country into a socialist one. And did it happen? Well obviously not, at least obviously to anyone not living in fantasy reds-under-the-bed land. --Demiurge1000 (talk) 07:42, 22 March 2011 (UTC)[reply]
Qadaffi has managed to build an impressive list of enemies in his 40-plus years in power, both in the east and the west, and it looks like it's payback time, i.e. he's presented an "opportunity" to his enemies, to take some action. People seem to have forgotten that Qadaffi's name was once mentioned in the same tone of voice that later came to be used for Saddam Hussein. Some idea of Libya's image in the eye of the American public at one time can be seen in the 1985 film Back to the Future, where they mention "Libyan terrorists". Not just "terrorists", not just "Arab terrorists", but specifically Libyan terrorists. Than Saddam came along and stole Qadaffi's thunder, so to speak. ←Baseball Bugs What's up, Doc? carrots16:36, 20 March 2011 (UTC)[reply]

Qadaffi has accomplished to re-build his image from terrorist to acceptable partner, just to lose all respect from the West when it came down to fight the rebels. Strangely, in his present list of enemies, Arabs are well-represented. That helps the West, since no one is going to accuse them of being Islamophobic. Quest09 (talk) 17:55, 20 March 2011 (UTC)[reply]

I suppose the presence of oil in Libya might anything to do with outside countries' interest in its governance. Operation Iraqi Freedom was purportedly called Operation Iraqi Liberation until someone realized the acronym didn't convey the message they wanted. 75.57.242.120 (talk) 04:49, 21 March 2011 (UTC)[reply]

In the case of Libya, since the oil supply was fairly regular under Qadaffi, it's reasonable to expect that it would be again, if everyone just let him massacre his enemies and regain control. A protracted civil war or new form of government, on the other hand, could both threaten the oil supply. So, the theory that the "Western interference is to gain control of the oil" doesn't make much sense. StuRat (talk) 15:55, 23 March 2011 (UTC)[reply]

World War 3

I often hear people mention how bad the world will be when World War 3 comes along... But in reality, we're already in it; the war of terrorism. It's world-wide, and causing a lot of damage. Wouldn't you agree that this is in fact World War 3?Accdude92 (talk) 04:14, 20 March 2011 (UTC)[reply]

I've never seen anyone apply that particular term to current events: the term usually applied is the War on Terrorism, and that's even debatable (many of the current series of conflicts are only tangentially related to terrorism). WWIII was usually applied, up until the breakup of the Soviet Union, to a potentially hot phase of the Cold War, which would have indeed been bad. Acroterion (talk) 04:19, 20 March 2011 (UTC)[reply]
No, and to continue to call it a war does the world no good at all. It will be as successful as the war on drugs. HiLo48 (talk) 04:20, 20 March 2011 (UTC)[reply]
Nope. We're already into World War 7 (I think - I've lost count). Seriously, though, do you think that terrorism is a new phenomenon? Look here for why you are wrong. AndyTheGrump (talk) 04:21, 20 March 2011 (UTC)[reply]
Not new, exactly, just a large issue now... More so than it appeared in the past.Accdude92 (talk) 04:34, 20 March 2011 (UTC)[reply]
Even that depends on where you are. In Northern Ireland (and probably Spain, and no doubt other places too), it is less of an issue than it used to be. In any case, 'terrorism' is such a vague concept that it tells you little about anything. As for terrorism causing 'a lot of damage', as the OP suggests, I think this is overstating things - look at what happened to Coventry, Hamburg, or Hiroshima, and at the death-camps and other mechanised mass-murder, and ask yourself if things are that bad? No, they aren't - though it suits some politicians (and arms manufactures etc) to suggest it is. AndyTheGrump (talk) 05:09, 20 March 2011 (UTC)[reply]
I was going to suggest that the 1970s must have seemed, to people at the time, to have been a hotbed of terrorism, more than the 2000s, due to its seemingly endless series of airline hijackings, the Munich massacre, the Symbionese Liberation Army in America, and the peak of violence in Northern Ireland (see The Troubles) ... but this page shows that indeed terrorism in Israel and in Iraq in the past 15 years, as well as the 9/11 attacks, has really dwarfed the 1970s incidents (according to what the page says is the RAND definition). But 4,500 deaths in a year doesn't compare to the staggering casualty figures of either World War. Comet Tuttle (talk) 18:16, 21 March 2011 (UTC)[reply]

You're not the first to think that the War on Terror is a world war. See Norman Podhoretz, for example, who wrote World War IV: The Long Struggle Against Islamofascism (the Cold War is his WW3). Almost everyone agrees now that the War on Terror was misnamed, even George W Bush. But to have called the war by a more accurate name, like the War against Extreme Militant Islamism, would have been even more controversial, so the euphemistic name was used. Whether this conflict is really a war, or really a world war, or really like the Cold War is debatable, and this isn't the place for that debate. —Kevin Myers 05:24, 20 March 2011 (UTC)[reply]

This isn't a question, it's an invitation to debate. 87.114.246.141 (talk) 11:01, 20 March 2011 (UTC)[reply]

Anyway, world war three was the one against Napoleon, they mean world war 6, and I am sure we will only know it was that once it finishes and we can look back and see just what we have actually achieved. Though, Churchill did say the empires of the future would be empires of the mind, and the opposing sides in this conflict seem to be just such predicted mental empires, with competing views, so maybe. 148.197.121.205 (talk) 14:25, 20 March 2011 (UTC)[reply]
As far back as 2002 or 2001, Bill O'Reilly (who typically calls it the War on Terror) was saying this was really World War III. The numbering came about because what we now call World War I was called "The Great War" or "The World War" (without a number, obviously) as well as the hopelessly optimistic "The War to End All Wars". Then when a truly world-wide war came along, they tacked a I onto "the" World War, and a II onto the "second" one. It was long assumed that World War III would be a mutually-destructive nuclear war, i.e. what would happen if the cold war went "hot". As Tom Lehrer said in 1965, "If any songs come out of World War III, we'd better start writing them now!" However, history has a way of evolving in different directions than expected. ←Baseball Bugs What's up, Doc? carrots15:48, 20 March 2011 (UTC)[reply]

In reality the term World War is more a description about a state of total war between great powers than the scope of such a conflict. There have been only two such conflicts since mechanised warfare was created, and we are not in the third. 130.88.162.13 (talk) 18:22, 20 March 2011 (UTC)[reply]

If 20 million people die, you can call it a world war. Otherwise it is just various seperate wars or even compared to WWII, skirmishes. --Lgriot (talk) 10:04, 21 March 2011 (UTC)[reply]
I guess it all boils down to agreeing on a definition of World War. Without having put too much thought into it, I am tempted to suggest at least two factors that should be present for a conflict to fall into the "World War" category:
(1) An armed conflict between official nations
(2) One or more nations of subsantial economic and military power on both sides of the conflict
Introducing an element of scale (ie. number of casualties and geographic spread) would also be helpful, but at least the proposed two conditions rule out concepts like wars against terrorism, aids, drugs etc.... --DI (talk) 11:14, 21 March 2011 (UTC)[reply]

I am looking for info on When and where was the first online legal notice served that was held tenable by the judge

Would appreciate any help —Preceding unsigned comment added by 76.31.252.236 (talk) 19:42, 20 March 2011 (UTC)[reply]

Quibble: The first online legal notice served that was held tenable (enforceable) by the judge was not necessarily, and I assume was not, the first online legal notice in the world. A giant majority of legal notices are never ruled on by any court, so I expect the first online legal notice was something relatively uncontroversial, like "Copyright (C)1970 by Knuth". By "online", also, you might want to specify whether you are talking about the Web, or on ARPANet, or over dial-up modem BBSs, etc. Comet Tuttle (talk) 20:23, 20 March 2011 (UTC)[reply]

I am referring to the web —Preceding unsigned comment added by 76.31.252.236 (talk) 20:55, 20 March 2011 (UTC)[reply]

So not the internet? The internet predated the web by a decade or two. --Demiurge1000 (talk) 22:16, 20 March 2011 (UTC)[reply]

Records management - Dutch law

Hi. I have a problem in finding answers to following questions regarding Dutch record and archive law: What information must be registered? (according to law and best practice) What kind of quality is demanded from this registration? What retention period applies? (search for laws or special retention lists) Who uses this information? (for instance: controlling and auditing agency’s)

I should consider those questions in respect to companies such as Energy companies, Telecommunications companies,Banks as financial institutions and public authorities. It is quite hard for me as I speak Dutch only a little. I found some English websites on that topic though,

http://www.dnb.nl/en/about-dnb/question-and-answer/questions-about-banks/index.jsp#faq-2 or http://www.thefreelibrary.com/Archives+and+records+management+in+the+Netherlands.-a057640379 however the don't really go into specifics.

If anyone has some knowledge on that subject or can point me in some direction (I'm not a law student) I would appreciate that. M2tek (talk) —Preceding undated comment added 21:24, 20 March 2011 (UTC).[reply]

For starters, Dutch wikipedia article Archiefwet (archive law): nl:Archiefwet, I don't have time to translate, but perhaps Google Translate can help. 93.95.251.162 (talk) 14:23, 22 March 2011 (UTC) Martin.[reply]
Law concerning retention duty of telecom data: nl:Wet bewaarplicht telecommunicatiegegevens .93.95.251.162 (talk) 14:45, 22 March 2011 (UTC) Martin again.[reply]
How long do companies need to store records for taxes? Site Dutch belastingdienst (tax inspectors):
http://www.belastingdienst.nl/zakelijk/ondernemen_administratie/ondernemen_administratie-02.html
Language Dutch, clicking on "English" in the upper left corner doesn't do much... 93.95.251.162 (talk) 14:35, 22 March 2011 (UTC) Martin.[reply]
Standards for keeping telecom traffic data:
http://tweakers.net/nieuws/71985/ministerie-ontwikkelt-standaard-voor-bewaren-telecomgegevens.html
93.95.251.162 (talk) 14:39, 22 March 2011 (UTC) Martin.[reply]
Dutch Data Protection Agency (language:English):
http://www.dutchdpa.nl/
See also: nl:College Bescherming Persoonsgegevens. 93.95.251.162 (talk) 14:54, 22 March 2011 (UTC) Martin.[reply]

Peaceful protesters arrested

http://www.washingtonpost.com/anti-war-activists-arrested-near-white-house-as-they-mark-8th-anniversary-of-start-of-iraq-war/2011/03/19/ABtZbzu_story.html

How can the government arrest peaceful demonstrators? Doesn't that violate the First Amendment rights of freedom of speech and assembly? --70.244.234.128 (talk) 21:47, 20 March 2011 (UTC)[reply]

See Free speech zone. As with all rights, there seem to be quite a lot of loopholes. --Mr.98 (talk) 22:03, 20 March 2011 (UTC)[reply]
We had this question the last time this happened, and the arrests were for refusing to stop "blocking the sidewalk." This appears to be the issue again this time. Protesters in this kind of case often seek to get arrested for the attention it brings to their cause. -- Mwalcoff (talk) 22:50, 20 March 2011 (UTC)[reply]
It should be noted that just because you are protesting something, you do not also get free reign to break reasonable laws, or in other words merely exercising your right (say, freedom of speech) doesn't grant you the right to violate one of my rights (say, freedom of movement) to do so. Police are allowed to disperse large groups of people who are preventing others from, say, walking down the street minding their own business. People who refuse to comply with a reasonable request (like "Hey, you're all clogging the sidewalk and making it hard for others to get to work. Break it up so others can get where they need to go") can be arrested for doing so. --Jayron32 23:02, 20 March 2011 (UTC)[reply]
In general, the trick to making this kind of thing First Amendment compatible is that you arrest whomever is doing it, whatever their political positions. --Mr.98 (talk) 00:54, 21 March 2011 (UTC)[reply]
And where do you find pro-war protesters to arrest as a compensation? 212.169.189.144 (talk) 01:06, 21 March 2011 (UTC)[reply]
You can find protesters for anything these days. --Mr.98 (talk) 02:13, 21 March 2011 (UTC)[reply]
Off-topic pedantry, but I can't resist - it's "free rein", not "free reign", being an allusion to horses, not rulers. AndrewWTaylor (talk) 11:34, 21 March 2011 (UTC)[reply]
"Once the dictator no longer reigned over them, they had free rein to do as they pleased, as the blessings of liberty rained down upon them." StuRat (talk) 18:56, 21 March 2011 (UTC) [reply]

This is part of the reason in the UK at least, a march is a better idea than a static protest. Under UK law, public rights of way must be used for the purpose of "passing and repassing" - in other words, on a strictly legal basis you must keep moving and not stop and block the right of way for others. Exxolon (talk) 19:45, 21 March 2011 (UTC)[reply]

From the Post report, it sounds like Mwalcoff has it right (though they may have been blocking the driveway rather than the sidewalk). Unfortunately the U.S. media usually makes a point of ignoring protests unless someone gets arrested, creating some motivation for protesters to get arrested for some minimal offense like this. However, the media coverage is often unreliable, and I know that protesters do get arrested on completely bogus charges when someone wants them out of the way. For example, at the 1996 Democratic convention they formally charged several leaders with bizarre crimes like a felony charge of throwing a plastic bottle and an animal rights activist accused of punching a horse. The way it actually worked was that a few hours before the president showed up, the cell phones all went dead and meanwhile the leaders were all rounded up. Eventually everything was dropped, but only after a long legal action that further drained their efforts. Wnt (talk) 22:57, 21 March 2011 (UTC)[reply]
As Mwalcoff and Jayron point out, while we have the right to peaceably assemble and protest, that does not mean anytime-anywhere. The protestors are not being arrested for protesting, but for infringing on the rights of others to use public right-of-ways. And certainly getting arrested (usually for disturbing the peace or whatever) is a good attention-getter. ←Baseball Bugs What's up, Doc? carrots23:08, 21 March 2011 (UTC)[reply]

AIMAV

What does AIMAV stand for? It's some sort of linguistic organization. Geschichte (talk) 22:23, 20 March 2011 (UTC)[reply]

I found a book review from 1971 that says this is a "merciful acronym" for the Association Internationale pour la Recherche et al Diffusion des Methodes Audio-Visuelle et Structuro-Globales. A later abstract says it is the Association internationale pour le developpement de la communication interculturelle, but that doesn't fit the letters. Looie496 (talk) 22:50, 20 March 2011 (UTC)[reply]

March 21

Two religious questions: "Come As You Are" and David's baby son going to heaven?

1. Where in the bible does it say "Come as you are?" (Regarding the clothes you wear to church.)

2. King David's son died as a baby as a punishment from God to David. I may have heard secondhand about his son going to heaven, but what verse indicates this?

Moreover, what other verses indicate that children before the age of accountability go to Heaven? What verses, if any, indicate that children could still go to hell? --70.179.169.115 (talk) 02:52, 21 March 2011 (UTC)[reply]

On your second question, the Old Testament, for the most part, does not mention reward and punishment in the afterlife. There are a few possible allusions to it in some of the later books (especially the Book of Daniel) but those are disputed. So most likely the question is not treated at all. --Trovatore (talk) 03:01, 21 March 2011 (UTC)[reply]
On question 2, after David's son's death, 2 Samuel 12:23 has David saying "But now [that my son] is dead, wherefore should I fast? can I bring him back again? I shall go to him, but he shall not return to me." It's up to the reader to decide if David's belief that he would "go to" his dead son means [1] in Heaven, [2] in the Bosom of Abraham, [3] in death, [4] in the ground, [5] in Sheol, [6] in Hell, [7] in the Limbo of the pagans, or [8] any of the really limitless other possibilities. Despite this nebulousness, those who wish to believe that dead babies go to heaven have found comfort in this particular verse. Historically, the beliefs regarding the "destination" of dead unbaptised babies have evolved over time—like most "Biblical" beliefs, verses can be picked out of context and interpreted in myriad ways, so that almost any doctrine can be justified, and has been. - Nunh-huh 03:56, 21 March 2011 (UTC)[reply]
Interesting; I didn't know that. That seems to supersede my answer for this specific question. --Trovatore (talk) 04:44, 21 March 2011 (UTC)[reply]
To answer the original questions — (1) The Bible doesn't say this. (2) This question is very much disputed; there's no specific passage that discusses the question in detail. Nyttend (talk) 11:19, 21 March 2011 (UTC)[reply]
Jewish belief is that pretty much everyone eventually goes to Heaven. Sinners may have a stop in hell first (maximum of 11 months). Children under the age of majority (12/13) are not responsible for their transgressions so would not go to hell. Ariel. (talk) 08:06, 22 March 2011 (UTC)[reply]
You are not alone, many people think "come as you are" is in the bible somewhere. Here's a list of passages that are relevant, some moreso than others [6] SemanticMantis (talk) 14:56, 21 March 2011 (UTC)[reply]
From the Christian Faith; Matthew Ch.2 16-18 may help with an answer. Heaven was only opened up after the Resurrection, see Harrowing of hell. MacOfJesus (talk) 23:02, 22 March 2011 (UTC)[reply]

About kinsmen relations of Vladimir Sviatoslavich the Great to Haakon Sigurdsson

I need help. Please, send to me link to source of info about kinsmen relations of Vladimir Sviatoslavich the Great to Haakon Sigurdsson. I will high appreciate your answer. http://en.wikipedia.org/wiki/Vladimir_I_of_Kiev

Oleg L.Gubarev <E-mail address removed to prevent spamming> —Preceding unsigned comment added by 84.52.73.176 (talk) 06:26, 21 March 2011 (UTC)[reply]

Monday the 22nd of 1826

What date in 1826 was a Monday the 2nd?--KAVEBEAR (talk) 08:23, 21 March 2011 (UTC)[reply]

According to our article on 1826, the calendar is here. The title of this thread differs from the question you asked. Monday the 22nd was in May. Monday the 2nd was in both January and October. Ghmyrtle (talk) 08:38, 21 March 2011 (UTC)[reply]
Here's a nifty site that will display a calendar for any year:[7]Baseball Bugs What's up, Doc? carrots11:07, 21 March 2011 (UTC)[reply]

World War II Air Raid sirens

On YouTube, I have been listening to recordings of both British and German air raid sirens which were used in World War II. My question is why were they different from one another? The British used a dual-tone siren while the Germans employed one with a single tone. I would have thought they'd have been standard.--Jeanne Boleyn (talk) 08:44, 21 March 2011 (UTC)[reply]

The British sirens were dual tone for an air-raid warning and single tone for the "all clear". I would imagine that Britain would not have wanted to buy it's sirens from Germany of vice-versa! Alansplodge (talk) 09:03, 21 March 2011 (UTC)[reply]
My apologies if this is obvious to everyone else but why would you think that they'd be the same? Were both sirens made by the same company? Dismas|(talk) 09:09, 21 March 2011 (UTC)[reply]
No, but seeing as stop signs, traffic light colours, etc. are all standard use internationally, I had presumed air raid sirens would have sounded the same. I have probably been influenced in my thinking by the fact that the British air raid siren is used as a generic siren in all World War II films, irrespective of nation. I recall having seen a film about the war based in Italy, and the British siren was used.--Jeanne Boleyn (talk) 12:32, 21 March 2011 (UTC)[reply]
Instead of using traffic signs, you should perhaps instead compare them to the situation regarding sirens on emergency vehicles, which can also vary a lot from country to country. --Saddhiyama (talk) 12:44, 21 March 2011 (UTC)[reply]
I know that having lived in four different countries. The US emergency vehicle sirens appear to be much faster and frenetic-sounding than those in Europe.--Jeanne Boleyn (talk) 12:48, 21 March 2011 (UTC)[reply]
Long ago I was taking a physics class with a professor who had been a kid in Germany when the allies were bombing and the air raid sirens were sounding. At some point a police care came whizzing by, blaring its siren, very audible as the windows were open. He said that when he heard an American police car or other emergency vehicle, it gave him chills, as it took him back to that time. It was always my understanding (tell me if I'm wrong) that European emergency vehicle sirens have that two-tone aspect to them purposely to sound different from air raid sirens. ←Baseball Bugs What's up, Doc? carrots14:54, 21 March 2011 (UTC)[reply]
Possibly. However, UK emergency sirens are different from those used in over in continental Europe. Road signs weren't harmonised until the late 1960s. I remember learning one set for a test at Cubs and then having to learn a whole new set because they'd changed. You can see the old UK signs here. BTW, the old air-raid sirens were kept in the UK until the 1990s in case WWIII broke out. In London, they were also used to signal a major flood, before the Thames Barrier was completed. There was a test once in the 1970s and it was a very erie sound - made the hairs on the back of my neck stand up. Alansplodge (talk) 15:51, 21 March 2011 (UTC)[reply]
The story I heard was that a new set of British road signs were designed in the 1960s by a British artist, and then these were copied by countries all around the world. I think they didnt pay her or the government anything. Edit: see Road_signs_in_the_United_Kingdom#Anderson_Committee - far more than you'd ever want to know about them. 92.15.25.108 (talk) 16:06, 21 March 2011 (UTC)[reply]
See Siren (noisemaker). I thought the sound from the traditional British air-raid siren was due to compressed air blowing through a series of holes in a rapidly rotating disk - hence the lower tone as it rotated up to speed. 92.15.25.108 (talk) 16:10, 21 March 2011 (UTC)[reply]
Which links to Civil defense siren, a rather informative article. --Saddhiyama (talk) 16:14, 21 March 2011 (UTC)[reply]
The British siren is scary enough, but when I heard the German air raid siren, I was so terrified, I wanted to run out of the room. It's like an announcement of unavoidable death. Go listen to it. I expecte they were designed to alarm people just as the Stuka dive bombers had sirens to heighten the terror of the population under attack.--Jeanne Boleyn (talk) 16:33, 21 March 2011 (UTC)[reply]
The answer appears to be that British sirens had two rings of holes in the Siren disk or disc, while German ones only had one. 92.15.25.108 (talk) 16:41, 21 March 2011 (UTC)[reply]
"seeing as stop signs, traffic light colours, etc. are all standard use internationally" That is not completely true even now, and certainly wasn't between the UK and the rest of Europe until relatively recently (1960s). See Road signs in the United Kingdom, though sadly that article has no illustrations of the old-style signs. Some (not very good) examples here. AndrewWTaylor (talk) 21:26, 21 March 2011 (UTC)[reply]
That's interesting. A pity the images cannot be uploaded to the article. I am also curious as to which air raid siren was used in Italy during WWII. As I said before, war films tend to use the British for effect due to its recognisability.--Jeanne Boleyn (talk) 07:24, 22 March 2011 (UTC)[reply]
Old German WWII Air Raid Siren YouTube Here's the German siren.--Jeanne Boleyn (talk) 13:51, 22 March 2011 (UTC)[reply]
"The video you have requested is not available." ←Baseball Bugs What's up, Doc? carrots13:54, 22 March 2011 (UTC)[reply]
Try again. I fixed the link. Here's the British siren. British siren with All-Clear plus vintage footage of the Blitz--Jeanne Boleyn (talk) 13:58, 22 March 2011 (UTC)[reply]
Wow. Scary stuff. But it got everyone's attention. The British siren sounds similar to the typical sirens used in the US, which are tested once a month on a specified day, and are also used for emergencies, the most typical being tornado warnings. ←Baseball Bugs What's up, Doc? carrots14:08, 22 March 2011 (UTC)[reply]
They are both scary. The British one, although eerie, seems to tell people to hurry while there's still time; whereas the German one is more dreadful, as if it's saying: "Too late. Your time is up. Death is now overhead".--Jeanne Boleyn (talk) 14:20, 22 March 2011 (UTC)[reply]
The German one is definitely scarier, but maybe that's because I'm used to the American sirens, which are similar to the British. However, the German item says, "here you can see/hear only the low-tone unit with big 3 ports". Does that mean there's a piece missing? ←Baseball Bugs What's up, Doc? carrots14:24, 22 March 2011 (UTC)[reply]
Regarding police cars, here's a youtube of a German car,[8] which sounds similar to the typical (or stereotypical) police car sirens used across the European continent and the UK. Here's an interesting sequence from New York City.[9] American police have a variety of types of sirens, which tend to get plenty of attention. Here's a nifty video that shows the control mechanism for those different types of sirens.[10] We're assuming the guy making the video had permission. :) ←Baseball Bugs What's up, Doc? carrots14:32, 22 March 2011 (UTC)[reply]

Knowledge of politics outside the US

Americans (of course excluding experts) are more knowledgeable about British politics. Compared to this, they have little knowledge/interest on the internal politics of other European countries like France, Germany etc., even through these countries are major global powers and their economies are ahead of UK economy. Is there any sociocultural explanation for this? Does language have something to do with it or other reasons? --Reference Desker (talk) 14:42, 21 March 2011 (UTC)[reply]

Judging by the amount of Americans who post on these reference desks who seem to think the UK is a socialist state, I would say your first sentence bears little resemblance to reality. --TammyMoet (talk) 15:23, 21 March 2011 (UTC)[reply]
And, given the number of Americans who insist on changing UK spelling to American spelling in article about British subjects, they don't know much about the language either. HiLo48 (talk) 15:28, 21 March 2011 (UTC)[reply]
Let's be nice to each other now. My guess is that it's because of the Special Relationship. It works ther other way too; I know a lot more about US politics than I do about French. Also, I wouldn't have been able to tell you who the Prime Minister of Japan was until last week. Alansplodge (talk) 15:38, 21 March 2011 (UTC)[reply]
American politics is of interest everywhere in the world, simply because American politicians have the (real or potential) ability to wield enormous political, economic and military power anywhere in the world. That's why some American politicians generate almost audible sighs of relief around the world when they are elected, and others are viewed as, simply, terrifying. Ghmyrtle (talk) 17:11, 21 March 2011 (UTC)[reply]
If you (awkwardly) recast the original assertion as "Of the politics of non-US countries, Americans are most knowledgeable about British politics," then there may actually be a grain of truth to that. Orange Suede Sofa (talk) 15:46, 21 March 2011 (UTC)[reply]
I would say the fact that the US was once a British colony may have something to do with it. Just a shot in the dark, mind, but worth thinking about.--Jeanne Boleyn (talk) 16:28, 21 March 2011 (UTC)[reply]
And, related to that, the fact that a high proportion of Americans (higher than for any other country I'd guess - I haven't checked) have ancestry from, and often current relatives in, the UK, so that they may be more interested in what happens there. The language issue is probably most important - the language used in the UK doesn't (usually) need to be translated for US viewers. Ghmyrtle (talk) 17:01, 21 March 2011 (UTC)[reply]
Most Americans don't know that Britain has a three party system, or anything about British politics. Most Americans don't even know who the speaker of the house is, or who has the majority in Congress.AerobicFox (talk) 16:49, 21 March 2011 (UTC)[reply]
"Britain" doesn't actually have a three-party system. It may be broadly true that England does - though there are many smaller parties who win significant numbers of votes there - but the nationalist parties are at least equally important players in Scotland and Wales, and Northern Ireland (which may or may not count as being part of "Britain" depending on whether you're defining it as the island, or as the UK) has a (largely) entirely different set of political parties. Ghmyrtle (talk) 17:04, 21 March 2011 (UTC)[reply]
(ec) "Most Americans don't know ... anything about British politics." Have to disagree. UK Conservative Party comes second to the Republican Party that interests US conservatives, not Germany's CDU, French UMP, even though both CDU and EMP are conservative parties. Similarly, American liberals identify themselves with the Liberal Democrats. France and Germany have their own conservative and liberal parties, but most Americans don't even know those names. And I think Margaret Thatcher is probably the only foreign figure who still influences American conservatism. --Reference Desker (talk) 17:46, 21 March 2011 (UTC)[reply]

Probably because they are all pretty similar. 129.120.195.14 (talk) 17:22, 21 March 2011 (UTC)[reply]

Yes, like Sinn Fein and the Democratic Unionist Party.--Jeanne Boleyn (talk) 17:39, 21 March 2011 (UTC)[reply]
(addressed @ User:Reference Desker) If you'd like to do some more general research on this topic, one keyword often used in media theory is "cultural proximity" (no article on Wikipedia, though it is listed among "conditions for news" in our article on news values). It's a loosely defined term, but often refers to people's (and the media's) preference for products close to their own values, culture, language, history, ... ---Sluzzelin talk 17:56, 21 March 2011 (UTC)[reply]
To add some of my own anecdotal examples: I frequently listen to WBUR or other NPR radio stations online. While it is night in the US, they often feature British programs. So I'm getting British news while listening to an American station. I live in Zurich, and sometimes know more about certain local politicians in Germany than about local politicians in French-speaking Switzerland, because I read, watch, and listen to German media far more often than to French. Yet I'm still far more familiar with French and Italian politics than I am with Swedish, Spanish, or Greek politics, because I virtually never read media outlets in their languages. ---Sluzzelin talk 18:30, 21 March 2011 (UTC)[reply]
The vast majority of Americans have no knowledge of politics outside the United States, and many have little knowledge of politics inside the United States. Most can name the U.S. president, but surveys have found that a majority cannot name the people who represent their state or district in Congress or key members of the president's cabinet. Of the few Americans (5% ?) who can even name a foreign head of government, probably the best-known heads of government would be the leaders of Canada and the UK. That is partly because people on the northern border can receive Canadian media in their own language, and because a substantial part of the 5% who know anything either listen to the BBC on the radio or subscribe to The Economist, both of which are produced in the UK. Marco polo (talk) 18:56, 21 March 2011 (UTC)[reply]
We can blame this appalling ignorance on the piss-poor US educational system. Jesus wept!--Jeanne Boleyn (talk) 19:25, 21 March 2011 (UTC)[reply]
It would be interesting to see some real data on this, rather than guessing. My general experience is that Americans know more about "enemy nations" governments than "friendly" ones —I would suspect more to know who are the leaders of Iran, Russia, China, and North Korea much sooner than they would know the leaders of Canada, the UK, and France. But this is just more guessing on my part, mixed with anecdata. --Mr.98 (talk) 01:32, 22 March 2011 (UTC)[reply]

IndexUniverse

Has anyone heard of indexuniverse.com, the source for info on ETFs and indexes? I don't see an entry on them and their publications ETFR (Exchange Traded Fund Report) and Journal of Indexes and the data on the ETF industry they provide. — Preceding unsigned comment added by Nycsanfran (talkcontribs) 16:59, 21 March 2011 (UTC)[reply]

Joint security area at NK/SK border

Hi all. I have a question about the meeting room at the JSA at the Korean border. The room is cut in half by the border (so half in South Korea and the other in North). Apparently it's possible to visit one of these rooms from the South side. How that is coordinated with the North? Is there some agreement where the South can bring in visitors on one day and the North on the other? In a similar way, this photo [11] shows a South Korean soldier on the North Korean side of the room. Does anyone know what the arrangement between the North and the South is regarding this? Thanks. - Akamad (talk) 17:04, 21 March 2011 (UTC)[reply]

Have you read the article Joint Security Area? --Saddhiyama (talk) 17:26, 21 March 2011 (UTC)[reply]
Yeah, I've read that article, but it doesn't seem to answer my question. It says that "both sides may allow tourists to go inside the MAC Conference Room", but there is nothing about what the arrangements are regarding this. In other words, how do they avoid having North Korean tourists and South Korean tourists entering the room at the same time? Or worse yet, how do they avoid DPRK soldiers and ROK soldiers entering the same room at the same time. They must have some rules regarding this, but I haven't been able to find any reference to them. - Akamad (talk) 17:47, 21 March 2011 (UTC)[reply]
I vaguely remember reading somewhere that they do (might) enter at the same time, but are not permitted to talk. Presumably they would be escorted out if they (or the other side) did talk. I think it was in the context of that being one of the ways for each side to see the other (i.e. the closest that interaction gets). Ariel. (talk) 07:53, 22 March 2011 (UTC)[reply]

Attacking Libya without congress approval

How can the US - and possibly also its partners - attack Libya without congress approval? 212.169.185.194 (talk) 18:18, 21 March 2011 (UTC)[reply]

It was a United Nations resolution to attack Libya, not solely the USA's.--Jeanne Boleyn (talk) 18:27, 21 March 2011 (UTC)[reply]
The Constitution only requires Congressional approval when war is formally declared (Declaration of war by the United States). There are limits as to how long a conflict can go on, though, before Congress is consulted (War Powers Resolution). Technically Congress could probably de-fund the activity if they wanted to (power of the purse), though, additionally technically, the President could probably find ways to divert funds to it again (e.g. Iran–Contra affair, black budget). --Mr.98 (talk) 18:28, 21 March 2011 (UTC)[reply]
first point, other nations do not need the approval of the US congress to do anything.
other than that, the president is empowered to engaged in military actions of certain kinds without consulting congress. This is designed to allow rapid response to situations impacting on national interests, and includes acting on resolutions passed in the UN - no one wants something potentially damaging to the national interests to continue unimpeded while the members of congress debate and debate and debate over what to do. Only congress can explicitly declare war, of course, but that power has been weakening since the 1950s, when presidents started indulging in full-scale military 'police actions', using their power to engage immediate threats to commit to extended military involvement without declarations of war. --Ludwigs2 18:29, 21 March 2011 (UTC)[reply]
I suppose the OP meant congressional approval from their respective congresses, which could be granted or not, be necessary or not. Quest09 (talk) 18:35, 21 March 2011 (UTC)[reply]
If that's the question, then as far as the UK is concerned, what happened is that the government in power took the military action that it believed was necessary (having extensively trailed that decision over the last week or two thus allowing opportunity for Parliamentary comment) and then that action was discussed by Parliament at the earliest sensible opportunity, which I believe was today. This in theory gave Parliament the opportunity to condemn the government's action if they chose to do so. The exact mechanisms by which the discussion in question was put in place are down to the inner workings of Parliament (I think the government co-operated in making sure it happened), but there are many such mechanisms for different circumstances; for example Robin Cook used his resignation statement (billed in Parliamentary terms as a "personal statement") to make his acclaimed speech condemning the Iraq war (YouTube version). --Demiurge1000 (talk) 19:00, 21 March 2011 (UTC)[reply]
There is still no requirement on the British government to obtain the support of Parliament for military action; a White Paper issued early in Gordon Brown's premiership indicated that such a procedure would be made compulsory (see paras 25-30 here) but it is difficult to see how in the United Kingdom constitution it could really be made binding. Nevertheless, it remains the case that there is a political necessity on the Government to obtain Parliamentary approval. The debate yesterday was on a motion explicitly supporting "the use of UK armed forces and military assets in accordance with UNSC Resolution 1973" and was agreed with an overwhelming majority of 557 to 13. Sam Blacketer (talk) 11:39, 22 March 2011 (UTC)[reply]
@Jeanne: The UN does not have the authority to do that, since they do not represent the American people.
Officially, it is not a war declaration, nor a war. The US is just enforcing a UN resolution. Quest09 (talk) 18:35, 21 March 2011 (UTC)[reply]
I realise that the UN cannot force a nation to go to war, but I wanted to point out that it isn't an American-led enterprise as in Bush's Iraq War. Five nations so far have abstained from joining the coalition against Ghaddaffi.--Jeanne Boleyn (talk) 18:45, 21 March 2011 (UTC)[reply]
The UN cannot whether force nor authorize the US to go to war. "Bush's Iraq War" is ambiguous. Quest09 (talk) 18:55, 21 March 2011 (UTC)[reply]
I think "abstained from joining" is maybe potentially misleading. The five (Brazil and Russia and India and China and somewhere, if memory serves) merely abstained on the UN resolution. That of course also implies those five won't be providing military forces or logistical support as part of the coalition. However, I would guess it's entirely possible for a country to vote in favour of the UN resolution and not provide military forces or logistical support. Thus, supporting the resolution isn't necessarily the same thing as "joining the coalition" in my view, although it might be the same thing as "backing the coalition". --Demiurge1000 (talk) 18:52, 21 March 2011 (UTC)[reply]
You are completely right on this. 10 nations voted 'yeah, bomb them all', but 10 nations are not bombing Libya. Quest09 (talk) 18:59, 21 March 2011 (UTC)[reply]
The President of the United States is commander in chief of the U.S. military forces and can order them into action without consulting Congress, though the War Powers Resolution requires him to obtain Congressional approval for military action within 60 days. Similarly, in 1986, President Reagan ordered the bombing of Libya without Congressional authorization. Marco polo (talk) 18:48, 21 March 2011 (UTC)[reply]
"The President does not have power under the Constitution to unilaterally authorize a military attack in a situation that does not involve stopping an actual or imminent threat to the nation." Senator Barack Obama, December 20, 2007. —Kevin Myers 13:54, 22 March 2011 (UTC)[reply]
Perhaps the Obama administration decided that the Libya situation does in fact represent a threat to the U.S. (Or are you suggesting that the Obama administration is turning into Bush's third term?) ←Baseball Bugs What's up, Doc? carrots14:13, 22 March 2011 (UTC)[reply]
Perhaps Obama thinks Libya represents an "actual or imminent threat", or perhaps he has changed his mind about the powers of the presidency, or perhaps he doesn't remember what he wrote, or perhaps he is deliberately violating the Constitution. The second option seems most likely. But some other liberals have not changed their minds about the power of the presidency. Dennis Kucinich has even mentioned impeachment. —Kevin Myers 14:34, 22 March 2011 (UTC)[reply]
It puts liberals in the position of defending Qadaffi, which is an interesting situation. Kucinich, of course, is pretty extreme in his views, even for a liberal. But if such a notion gained traction, it would put the GOP in an interesting situation also - because if they were to support impeachment, they would be hard pressed to justify their support of the Bush actions, and those of future Presidents, especially Republican Presidents. ←Baseball Bugs What's up, Doc? carrots14:46, 22 March 2011 (UTC)[reply]
[12] --Reference Desker (talk) 15:41, 22 March 2011 (UTC)[reply]
And [13] --Reference Desker (talk) 15:45, 22 March 2011 (UTC)[reply]
Cite a source, please, bugs, for anything you just wrote. Comet Tuttle (talk) 16:46, 22 March 2011 (UTC)[reply]
Actually, the points made in Reference Desker's second link are along similar lines. ←Baseball Bugs What's up, Doc? carrots16:49, 22 March 2011 (UTC)[reply]
That's not a source for your claims. I have problems with your use of "liberals" in the first sentence, and "pretty extreme" in your second sentence. Cite sources, please. This is a reference desk, not your personal soapbox. Comet Tuttle (talk) 23:24, 22 March 2011 (UTC)[reply]
I suspect the Obama admin would focus on the word "unilaterally" in that sentence. The odds are he briefed somebody in Congress about it before it occurred (e.g. one of the select committees that deals with things like this). Whether "unilaterally" means "requires a vote" is unclear. The Constitution sure ain't clear on it, and the War Powers Act gives a lot of leeway. --Mr.98 (talk) 21:34, 22 March 2011 (UTC)[reply]
It's highly misleading to suggest opposing the resolution and its enforcement or arguing President Obama's actions violate the US Constitute means you support Qadaffi. Incidentally, the right wing Accuracy in Media [14] is also strongly opposed to Obama's actions [15] as they were to Clinton's actions on Kosovo Nil Einne (talk) 01:42, 23 March 2011 (UTC)[reply]
It is politics. If this attack on Libya was ordered by Bush, liberals would have opposed it. Since it is ordered by Obama, conservatives are opposing it. Similarly if Iraq War was initiated by a Democrat president, Republicans would have opposed the war. --Reference Desker (talk) 01:53, 23 March 2011 (UTC)[reply]

Paul Trevor

I was wondering why there was no article or information about the street photographer, Paul Trevor? —Preceding unsigned comment added by 92.24.221.61 (talk) 19:15, 21 March 2011 (UTC)[reply]

Maybe because no one wrote it yes? At the first glance, he could pass the notability threshold. Quest09 (talk) 19:20, 21 March 2011 (UTC)[reply]

Moses

Hey guys, my lil sis needs help with her R.E. homework and I'm in the rather embarrassing position of not knowing the answer! It has a few questions about Moses, most of which we've worked out together, but we're unsure of the last two. It asks "how did his Hebrew background prepare him for his future role?" and "how did his Egyptian background help him...?" Now my knowledge of the Moses story is not that extensive, but as I see it the fact that he was brought up by Egyptians would make him more likely to be able to speak to the Pharaohs, since i imagine that the Hebrews, treated as second class citizens, would not be able to have access to them. His Hebrew background would make his people trust him more and believe in what he was saying, since he was one of them. But these two points fleshed out would only take up about 4 sentences, when she would probably need a couple o' paragraphs (she's only 11). Any help much appreciated! Postrock1 (talk) 19:59, 21 March 2011 (UTC)[reply]

Well I think you're right, but what point in his life are we talking about? Is it, for example, when he is being brought up in Pharaoh's household? Or when he is leading the Children of Israel across the Red Sea? Anyway, try and relate his dual nationality to the later events in his life (it is unlikely, for example, that God would have given the Ten Commandments to an Egyptian non-Hebrew) and you should get more flesh on the bones you already have. --TammyMoet (talk) 20:51, 21 March 2011 (UTC)[reply]
That's the thing, we're not 100% sure since she missed a few days off school. The previous question was asking about his excuses and indecision in following God's word, which I think is referring to when he went to the pharaohs to tell them to free the isrealites, but at first he's too scared too? I think. haha I actually have no idea about this, I would fail on Are You Smarter Than A 10 Year Old Postrock1 (talk) 21:17, 21 March 2011 (UTC)[reply]
Has she got any friends in that same class? Ask to borrow their notes, or for them to help summarize what the topic is. — The Hand That Feeds You:Bite 21:51, 21 March 2011 (UTC)[reply]
The question can be answered here - but can it be answered to the satisfaction of the class's particular religious interpretations? For example, I might say that Moses could hardly have presented himself as a Hebrew prophet without the appropriate background. Or I might argue that if he hadn't had intimate familiarity with the plans and construction of the Ismailia Canal, he couldn't have led a horde of Israelites down it with the Egyptian Army on their heels, arranging for his comrades to close up the locks and dry up the flow of that branch of the Nile long enough for them to cross it, opening them back up afterwards and swamping the Egyptian chariots. But I might get a big fat F for that one. ;) Wnt (talk) 23:07, 21 March 2011 (UTC)[reply]
"... his training as a member of Pharoh's household had doubtless given him dignity, confidence, poise, and had accentuated his ability to organize and command..." Truly Moses had a diverse background, but like an athlete or musician with natural ability, these qualities must be cultivated. That is a big part of the reason that Moses had to undergo another 40 years of training: "the qualities of patience, meekness, humility, long-suffering, mildness of temper, self-control, and learning to wait on Jehovah needed to be developed in him to a higher degree, in order for him to be the fitting one to lead God's people." (Insight on the Scriptures, v. 2, pg. 435). Hope this helps! Schyler (one language) 00:01, 22 March 2011 (UTC)[reply]

Questions like that are really designed to make a student think, not to have definite answers. Doing the thinking for her basically defeats the purpose of the exercise. Looie496 (talk) 04:37, 22 March 2011 (UTC)[reply]

Read Exodus chapter 2 in a decent translation. Rashi's commentary is helpful, too. If you'd like to score well, don't overlook Moses' sojourn with Jethro in Midian, which helped form his character and is neither necessarily "Hebrew", nor "Egyptian". The fact that he's initally mistaken for an Egyptian is also interesting. --Dweller (talk) 15:47, 22 March 2011 (UTC)[reply]

It sounds like you're having trouble remembering the story in detail, but don't have a copy to refer to. Did you know there are many online Bibles available, including in fairly easy-to-read English? For example... 212.183.128.13 (talk) 19:04, 22 March 2011 (UTC)[reply]
I suggest read the relevent texts to her, explaining in simlpe English as you go along and the answers should appear naturally. Moses was au fait with the house of Pharaoh, and there were not many men around of his age as Pharaoh had ordered their deaths at birth. MacOfJesus (talk) 00:04, 23 March 2011 (UTC) Also, Moses had run away from Pharaoh over the death of an Egyptian. God ordered him to go to Pharaoh. MacOfJesus (talk) 00:26, 23 March 2011 (UTC)[reply]

Fringe benefits: US Health care

Hello. Me again!

I've learned that US employers provide improved health insurance as a fringe benefit that is not taxed. This apparently goes for 20% of US employers - or something like that. Now, Donald Barr claims that when you get (after discussing with your employer) an insurance that is better than the last - covers more stuff etc - the HMO's costs go up. That's what's problematic to the feds about the fringe benefit insurances. How does this happen? The only way I see prices going up is by the employee showing up at the doctor's office/hospital, and receiving MORE care for an injury of one type, than he would with the SAME injury on a lesser insurance. Ie, that the physician would feel it prudent to take more tests, each and every one with less actual reward for the bucks spent. Since this sounds like a very difficult thing to measure, I thought I'd ask if it was the case.

Thank you in advance. 80.213.11.105 (talk) 23:23, 21 March 2011 (UTC)[reply]

The argument is not that these plans drive up costs for the HMOs; it's that they drive up the cost of healthcare in general by shielding consumers of healthcare from any of the costs that might reduce demand. Remember that in the US system, the government does not impose any limits on healthcare providers, pharmaceutical companies, and so on, who are free to charge what the market will bear. If consumers of healthcare are shielded from price constraints, then there is potentially no limit to healthcare costs. See this article for more information on this topic. Marco polo (talk) 23:29, 21 March 2011 (UTC)[reply]
I wish to clarify: When I ask what the concrete consequences are that up the prices of healthcare, I propose that it is the inclusion of more tests, or slightly more costly treatment, by the physician - OR, by simply seeing the doctor more often. Your link seems to echo this, "... the most expensive plans — which some argue encourage overuse of medical care...". What you are saying, I think, is that one must view the problem as one based on supply and demand. Now, here's where I lose you: Are you saying that if presented with an insurance plan detached from the employer (thus based on your own desires for this plan), then that plan would necessarily involve cheaper treatment? I do not see the connection between increasing the profit for a HMO by buying unnecessarily expensive insurances and an increase in federal expenses, without receiving a more expensive treatment/drugs for the same illness. That, and seeing the doctor more often - but also then receiving more treatments. What I hear from a lot of people is that the prices for drugs skyrocket because the feds are by and large conned into covering them (the demand you speak of). What this entails to me is that there must be one treatment that works OK and is cheaper, and another treatment that works OK and is more expensive. 80.213.11.105 (talk) 00:48, 22 March 2011 (UTC)[reply]
Right. I think one of the distinctions of the most generous plans is that employees who receive them do not contribute to their cost and do not face any fee when they go to seek treatment. So people with those plans will not think twice about continuing to attend physical therapy sessions every week for six months at $300 per session even though they received 95% of the benefit of the sessions during their first month of attendance. Or, people with those plans may wish to have a $2,000 MRI scan every time they experience the least back pain. By contrast, my own US employer requires me to pay 15% of the cost of the insurance, which gives me an incentive to choose a plan with some restrictions and limits, since it is less expensive to me personally. All plans offered by my employer also require the employee to pay a set fee, or "co-pay" for every visit to a healthcare provider. So, for example, when I received physical therapy, I had to pay $30 for each session. This covered only a fraction of the session's cost. However, once I felt that the benefit of attending an additional session wasn't worth another $30, I canceled the sessions. If I had a plan without fees or limits, I might have kept going, since the physical therapy was pleasant in and of itself. Finally, you mention one treatment that is okay and cheaper and another that is okay and more expensive. This is precisely the case, in many cases, with brand-name versus generic drugs. My plan requires me to pay a much higher fee for a brand-name drug if a generic version of the same drug is available, so I will almost always choose the cheaper generic. People with insurance policies that don't penalize them for choosing the more expensive version are more likely to do so. Marco polo (talk) 01:24, 22 March 2011 (UTC)[reply]
Thank you, Marco! You've been very good help to me. 80.213.11.105 (talk) 16:40, 22 March 2011 (UTC)[reply]

March 22

What happened to this house?

House in severe disrepair

Unfortunately, the historic house in this picture is in severe disrepair, despite its designation as a historic site by the US federal government. While I can understand what's caused most of its problems, I'm puzzled by the darkness around the chimney: what could have caused it? Fire was my first thought, but I can't imagine fire coming out of the brickwork just below the chimney without being severe enough to scorch the walls around the windows and doors. I also considered that it might have been burned but that the other damage was repaired; however, given the state in which the house sits, I doubt that anything has been repaired anywhere near recently. Finally, please note that I'm somewhat colorblind, so I might be unable to see something that would be obvious to most people. Nyttend (talk) 04:08, 22 March 2011 (UTC)[reply]

One thing that is obvious is that the chimney is a later addition to the building; the brickwork of the chimney is distinctly pink, while the rest of the house is orange brick. My guess is that the staining is from an older, long gone chimney that the current one replaced. Probably, the first chimney stained the wall, was removed, and the newer unstained chimney put on in its place. --Jayron32 04:16, 22 March 2011 (UTC)[reply]
Yes, that makes sense. The staining on the wall below the chimney looks to me like damp penetration. Perhaps the original chimney had been so badly damaged it was replaced? There is a crack in the brickwork to the right of the chimney, but that looks more like settlement damage than anything related to the chimney. Actually, brick buildings have a surprising tolerance for settlement etc, if they stay reasonably watertight - I'm living in a 100+ year-old house with multiple cracks, and the roof held on solely by gravity, from what I can tell (and some dubious brickwork at the back of the house where it isn't so noticeable), but I don't think it is going to fall down any time soon - I'd be more worried about internal floors etc in the building shown. If the floors ans/or roof trusses are rotten, that is likely to bring the building down more quickly than problems with brickwork. AndyTheGrump (talk) 04:33, 22 March 2011 (UTC)[reply]
Andy, is it possible that we're sharing the same house and haven't noticed? {The poster formerly known as 87.81.230.195} 90.201.110.155 (talk) 14:23, 22 March 2011 (UTC)[reply]
It's quite possible to have a fire inside a chimney -- soot can build up and eventually ignite, sending flames all the way up the shaft. If it lasts long enough, the bricks can get pretty hot. Looie496 (talk) 04:36, 22 March 2011 (UTC)[reply]
I'm not so sure about a fire - I recall seeing old British houses with similar markings near the chimney. 92.15.6.157 (talk) 11:37, 22 March 2011 (UTC)[reply]
In my 1960s childhood in London, chimney fires were a common occurence. The fire brigade would put them out with a stirrup pump attached to a lomg tube (in sections I believe) which they would poke up the chimnety. Alansplodge (talk) 12:25, 22 March 2011 (UTC)[reply]
Back in the 1980s, my chimney in Dublin would often catch fire due to my bad habit of burning coal and peat together. Rather than risk the Fire Brigade flooding my house, I'd put it out myself by stuffing wet newspapers up the chimney.--Jeanne Boleyn (talk) 13:45, 22 March 2011 (UTC)[reply]
Why would a fire mark the chimney so far down from where the smoke comes from? It may be due to the volatile components of the smoke condensing on the comparatively cool inner surface of the chimmney and leaching through the brickwork. In other words, its tar. If it was damp or mildew from rainwater running down the side of the chimney, as speculated above, then wouldnt the lower part of the wall of the most exposed part of the house be like that?
The chimney bricks may be exactly the same as the other bricks, except they are much more weathered due to their exposed condition, including being exposed on both sides of the briock. You can see some visible efflorescence on the chimney, and in addition the surface may have spalled off due to freezing while damp. There is likely to be mold or moss growing due to the unevenness of the surface, which needs repointing. You can see some partial repointing has been done. All of the preceeding things result in colour changes. 92.15.23.133 (talk) 18:12, 22 March 2011 (UTC)[reply]

I agree that the chimney looks like it was rebuilt with different colored brick. It is very likely that the old chimney crumbled and there was water damage to the brickwork below it. What is obviously a big problem is the collapse at the left. The windows are broken, and the old copper roof might have leaked near the peak, allowing in rain, leading to collapse of rafters (causing roof collapse and allowing in more rain), leading to rot and collapse of floor joists and studwalls. The collapse of floor joists often brings down outer brick walls due to the joist end acting as a lever on the brickwork above it. Joist ends were sometimes tapered and set in pockets in the brick to allow them to collapse while sparing the wall. If cost were no object, it could probably be rebuilt. The White House, for instance, was gutted and rebuilt in 1949-1951, while preserving only the roof and outer walls. The walls in general look to need tuckpointing if the collapsed section were rebuilt. Once rain starts entering an old building due to bad windows or roof, ruin is only a few years away. Edison (talk) 21:05, 22 March 2011 (UTC)[reply]

Purpose of an elaborate wedding

Why is the forthcoming marriage of William Windsor and his girlfriend being staged in such an elaborate manner? Wouldnt a quiet wedding at the Windsor chapel or registry office be better; and it could still be covered on tv? I am asking from a sociological point of view. Thanks 92.15.6.157 (talk) 11:35, 22 March 2011 (UTC)[reply]

I'm surprised you think it's elaborate! Certainly compared to his father's first wedding it's not. From personal experience, the first wedding tends to be the most elaborate (I refer to mine as my "meringue moment" because of the white lacy dress I wore), whereas subsequent weddings tend to be less elaborate. As this is the first wedding for either party, it's more elaborate than, say, his father's second wedding. --TammyMoet (talk) 11:53, 22 March 2011 (UTC)[reply]
Because it's what we Brits call a royal wedding, which is a great opportunity for a PR exercise.--Shantavira|feed me 12:48, 22 March 2011 (UTC)[reply]
All royal weddings (Charles and Camilla notwithstanding) tend to be extravagant; the public would be disappointed if they were simple affairs, without pomp and ceremony.--Jeanne Boleyn (talk) 13:42, 22 March 2011 (UTC)[reply]
For some of the UK public that's probably true - others just wish everyone would shut up about it. Although much of the pre-publicity is completely over the top and vomit-inducing, I'm still hopeful that attitudes have changed to the extent that there won't be quite as much obsequiousness and time wasted on this one as there was for his dad's first wedding, or for his granny's various "jubilees". Harrumph. Ghmyrtle (talk) 16:05, 22 March 2011 (UTC)[reply]
How many anti-British monarchy posts do we get each week? —Preceding unsigned comment added by 88.6.114.190 (talk) 13:50, 22 March 2011 (UTC)[reply]
Not enough. It's still there. Ghmyrtle (talk) 16:08, 22 March 2011 (UTC)[reply]
And I can't imagine opinionated bores venting their spleen on the Wikipedia Reference Desk are likely to change the status quo anytime soon. But thanks for sharing. 87.114.246.141 (talk) 20:28, 22 March 2011 (UTC)[reply]
Annoying thing is when it comes to these anti monarch posts it's nearly always the same OP who starts it Nil Einne (talk) 01:46, 23 March 2011 (UTC)[reply]
The aame could be asked of any wedding, not just a royal one... 207.81.30.213 (talk) 14:32, 22 March 2011 (UTC)[reply]
The more pomp and circumstance, the more souvenir videos they can sell. ←Baseball Bugs What's up, Doc? carrots14:34, 22 March 2011 (UTC)[reply]
I was about to post the same thing: pomp and circumstance. Also, decadence. A decadent society loves a decadent spectacle. Vranak (talk) 17:02, 22 March 2011 (UTC)[reply]

What is its purpose from a sociological point of view please, compared with a normal wedding? 92.15.23.133 (talk) 17:28, 22 March 2011 (UTC)[reply]

How do you know it's sociologically different from a normal wedding? Weddings are often big extravaganzas. Just more so with a royal wedding. A wedding gala of any kind is supposed to be a celebration of a significant life event. ←Baseball Bugs What's up, Doc? carrots17:33, 22 March 2011 (UTC)[reply]
Oh, it is sociologically different all right. We don't get a day off work to celebrate most weddings. Is this question for your sociology homework? If so, you should mention the mass media, and popular culture, in particular celebrity culture. Why didn't Jordan and Peter Andre, or the Beckhams, have quiet weddings? But it seems that you are taking a functionalist viewpoint, assuming that the wedding must have one purpose. You might get further with an interpretivist perspective. Does the wedding have the same meaning for all social groups? Itsmejudith (talk) 17:45, 22 March 2011 (UTC)[reply]
That's an interesting question. I don't have the answer, but note that elaborate weddings seem to be far more important to women than to men. Perhaps it's a way of establishing a woman's social standing (while a man's comes primarily from his job) ? It might be interesting to compare with working women, to see if they choose less elaborate weddings. StuRat (talk) 17:46, 22 March 2011 (UTC)[reply]
People seem to go back to more conservative gender roles in their wedding plans, compared to the roles they adopt in everyday life. Just basic things like a woman who wears jeans to work will wear a long dress for her wedding. There's something about a rite of passage that makes us want "all the trimmings". And of course it is a family occasion, so the couple want to be sure that the ceremony meets the expectations of both sets of parents. Within traditional values, women are supposed to care about their weddings. If they don't care, then the easy solution is not to bother with a wedding. Itsmejudith (talk) 17:55, 22 March 2011 (UTC)[reply]

Bread and circuses HiLo48 (talk) 17:53, 22 March 2011 (UTC)[reply]

I suspect this would be easier to understand in a more traditional society. When you read news about people caught up in bombings or riots in the Middle East it sounds like they're always on the way to a wedding somewhere. I suppose it was the same way elsewhere at some time. It seems to be the way for a family to be known, for people to form a sort of tribal identity, maybe even something akin to a potlatch economy. I may be far off the mark, but still, look at a palace. A royal can't sleep in 87 beds, no matter how fancy they are. Their whole business involves surrounding themselves with a circle of associates. Wnt (talk) 20:03, 22 March 2011 (UTC)[reply]

Pomp vs. pompousness

The above is proof, if proof were ever needed, that many people absolutely love pomp. They can't get enough of it. They'll travel half way around the world to see some. But individuals who are pompous are at the extreme other end of the desirability scale. Why? -- Jack of Oz [your turn] 20:31, 22 March 2011 (UTC)[reply]

Pomp and pompousness are two entirely different things. Pomp is a kind of public performance in which some are the principal performers, the majority both take minor roles and/or enjoy the spectacle as an audience, and everybody understands that they are all taking part in a kind of game. Pompousness/pomposity/pompous behaviour is merely the display of out-of-place and unwarranted arrogance. {The poster formerly known as 87.81.230.195} 90.201.110.155 (talk) 11:05, 23 March 2011 (UTC)[reply]
OK, let me rephrase the question. If pomp is generally considered a positive thing in its place, why was it used as the core of a word denoting a decidely negative quality? -- Jack of Oz [your turn] 11:11, 23 March 2011 (UTC)[reply]

Foreign deaths in Japan earthquake/tsunami

The deaths of vacationers was a large story in the Indonesian earthquake/tsunami and deaths of forigners made up 1% of the all deaths. So far I have heard of a single death in Japan of a foreigner (1% of 1%). Have there been more but the reporting buried in the other issues - or is this region of Japan too far off the tourist trail? —Preceding unsigned comment added by 75.41.110.200 (talk) 15:43, 22 March 2011 (UTC)[reply]

Deaths of foreigners have been reported, usually of those who have been working in Japan. Here in the UK there has been 2 British deaths reported, both ex-pat workers. --TammyMoet (talk) 15:48, 22 March 2011 (UTC)[reply]
One US citizen death was reported on the news last night. Corvus cornixtalk 17:49, 22 March 2011 (UTC)[reply]
The Australian government has declared that all Australians who they were aware of being in the danger area have been accounted for. HiLo48 (talk) 17:51, 22 March 2011 (UTC)[reply]
I found a report of two Filipino deaths[16] but can't find a story on any UK ones. Rmhermen (talk) 17:53, 22 March 2011 (UTC)[reply]
My bad. I tried to find the BBC report I remembered seeing last weekend, but couldn't find it, only a reference to the number of UK casualties being unknown. Either the story's been pulled, or I misinterpreted a report. --TammyMoet (talk) 18:36, 22 March 2011 (UTC)[reply]

Go back to ... (China, Africa, Whatever)?

What kind of people hear that most? People who look foreigner or those who speak with a foreign accent? Quest09 (talk) 17:01, 22 March 2011 (UTC)[reply]

"Go back to Africa": certainly, you only hear that if you are black. "Go back to China": all that look oriental, including Korean and Japanese could hear that. No, also not accent related. In Germany, also not accent related, I also heard the "go live in Turkey if you don't like it here" applied to an Arab. 80.58.205.34 (talk) 17:35, 22 March 2011 (UTC)[reply]
The "if you don't like it here" part is interesting. There is plain old xenophobia, where people just don't like foreigners, which seems rather irrational, then's there's the objection to foreigners who then want to change the new land to be the same as where they came from. This is certain to cause resentment from natives, for quite rational reasons, and does bring up the question: "Why did you leave your homeland, if you prefer everything the way it was there ?". StuRat (talk) 17:42, 22 March 2011 (UTC)[reply]
Sometimes, that's a change for better. A Canadian living in the US could claim their health system is better, and that the US should catch up. An American living in Cuba could demand a right to free speech. (although it might not be very wise). Quest09 (talk) 17:58, 22 March 2011 (UTC)[reply]
But doesn't it seem rather stupid of an American to move to Cuba and then complain about lack of free speech ? Why not just stay where he had it ? StuRat (talk) 18:00, 22 March 2011 (UTC)[reply]
This quickly got off topic, but there are good and bad things about every place, and people like to think the grass is always greener, so the above behavior isn't unheard of — Lee Harvey Oswald defected to the Soviet Union, which made front page news in some places, since he was an ex-Marine; but then moved back, unfortunately, when he decided he didn't like it there. Comet Tuttle (talk) 18:49, 22 March 2011 (UTC)[reply]
In Charleston, SC, there is a rather popular "Go back to Ohio" organization. The local minor league baseball team has even had "Go back to Ohio" nights. Mostly, you just see bumper stickers and an occasional ad in a local free paper. -- kainaw 19:05, 22 March 2011 (UTC)[reply]

In Australia there is a low level debate about changing our flag. There is a class of people known as Bogans, who tend to drive overly large four wheel drive vehicles with lots of stickers, including one showing the current flag, and saying "If you don't love it, leave." On a more international level, I have to ask, has "Yankee go home" faded from popularity? HiLo48 (talk) 19:36, 22 March 2011 (UTC)[reply]

Heh. I recently had a debate with a UK-based editor about British imperalism. His position was that if I had any objections, I should leave Australia. But I would not be welcome in his country. I'm still wondering where exactly it is I should be migrating to. -- Jack of Oz [your turn] 20:26, 22 March 2011 (UTC)[reply]
Yes. Not surprisingly, some of my Aboriginal friends are among those who would like to see a new flag. I'm really not sure where in the world they are expected to go if they leave. HiLo48 (talk) 21:55, 22 March 2011 (UTC)[reply]

LA as a place with beautiful buildings

I hardly thought as LA as a place with beautiful buildings. However, some (Hollywood) films challenged this assumption - (500) days of Summer and In Search of a Midnight Kiss, namely. How prevalent is worth-seeing architecture in LA? Quest09 (talk) 17:41, 22 March 2011 (UTC)[reply]

The interior of the Bradbury Building is used frequently in movies. The Griffith Park Observatory is often used. There are actually quite a few unusual buildings in the LA area. Corvus cornixtalk 17:51, 22 March 2011 (UTC)[reply]
For buildings older than 50 years, National Register of Historic Places listings in Los Angeles County, California and National Register of Historic Places listings in Los Angeles, California have a good selection. There's a good amount of Googie architecture in LA too, but it's fair to say that much of the LA area is an architectural wasteland. A big exception is Pasadena (National Register of Historic Places listings in Pasadena, California, with the Bungalow Heaven and the Gamble House. The hills contain a lot of California Modern architecture, a topic that WP apparently lacks. There's also some novelty architecture. Acroterion (talk) 21:11, 22 March 2011 (UTC)[reply]

Oldest historic human

Who is the oldest person who we know really existed, for which we have a historic record ? I mean to exclude both unknown human bones we've dug up and "historical people" who may well be simply legends. StuRat (talk) 18:06, 22 March 2011 (UTC)[reply]

Walter Breuning is currently the world's oldest, at 114 or so, and Jeanne Calment was the oldest verified, at 122. In the "see also" there are lists of the oldest verified. Obviously, Biblical tales about guys like Methuselah can't be independently verified. One legend is that Methuselah was the reason Midas got out of the medical field. They had lifetime guarantees on their hip replacements, and he came back for his free replacement so often that they went bust. ←Baseball Bugs What's up, Doc? carrots18:19, 22 March 2011 (UTC)[reply]
Oops, I see a clarification is needed:

I mean the dead person born the most years ago, like Moses or Abraham, or whoever we are certain really existed. I don't care how about the age they lived to be. StuRat (talk) 18:25, 22 March 2011 (UTC)[reply]

We've had questions like this before, I'm sure there was one recently...all I can find is a question about the earliest recorded event, but there must be others... Adam Bishop (talk) 18:28, 22 March 2011 (UTC)[reply]
(ec) Here's a thread from 2008 answering this. In it, Sluzzelin in turn invoked a 2006 thread. Comet Tuttle (talk) 18:29, 22 March 2011 (UTC)[reply]
Wow. Almost precisely the 3 year anniversary. ←Baseball Bugs What's up, Doc? carrots18:33, 22 March 2011 (UTC)[reply]
(ec)King Tut and his fam lived around 1300 BC, in their condo made of stone-a. ←Baseball Bugs What's up, Doc? carrots18:32, 22 March 2011 (UTC)[reply]
Bugs, if you'd read any of the previous threads, or even had a small amount of knowledge about ancient history, you would see how blatantly wrong your answer is. In ancient Egypt, kings are known are known by their name (or rather, inscription/heraldry) back to 3000 BCE (i.e. Hor-Aha) or so, about 1500 years before the start of the Eighteenth dynasty of Egypt, during which Tutankhamen reigned. Sumarrien kings as early as Enmebaragesi have been verified to actuctually have existed, more than 1000 years before Tutankhamen. Please supply references for your assertians, as you are less likely to give such absurdly wrong answers. Buddy431 (talk) 20:42, 22 March 2011 (UTC)[reply]
Are you channeling Comet Tuttle today??? ←Baseball Bugs What's up, Doc? carrots21:53, 22 March 2011 (UTC)[reply]
If you restricted yourself to answering questions you actually know something about, you wouldn't get snapped at like this. 87.114.246.141 (talk) 22:02, 22 March 2011 (UTC)[reply]
I don't think Buggs' answer was intended to be taken seriously. It was pretty clearly an SNL reference.
APL (talk) 23:06, 22 March 2011 (UTC)[reply]
Actually, I was just starting to fish for information on the oldest, and King Tut was the first old guy that came to mind. There was discussion on the ref desk talk page the other day, that editors shouldn't take drive-by shots at other editors in front of the OP's, but rather should take such matters to the ref desk talk page, for example. I'm guessing Buddy, and for sure the IP-with-4-total-edits, didn't get that memo. ←Baseball Bugs What's up, Doc? carrots00:31, 23 March 2011 (UTC)[reply]
Suggesting King Tut as the oldest known person is a gross error, and needed to be responded to here, so that people don't actually think that you have something worthwhile to say about this topic and believe your answer. I have posted my concerns on your talk page (because it's your issue, not the reference desk's), and you've made it disappear. If you don't want to discuss this there, where would you like to discuss it? I don't think the reference desk talk page is the best place (more drama than I like), but if you'd prefer, we could hash it out there. My talk page would also be acceptable, if you'd like to drop me a note. This answering with answers-pulled-out-of-your-butt business needs to stop. Buddy431 (talk) 03:14, 23 March 2011 (UTC)[reply]
I've had quite enough of your personal attacks tonight. I expect it from certain others, but I had thought you were above that sort of thing. Until now. ←Baseball Bugs What's up, Doc? carrots04:14, 23 March 2011 (UTC)[reply]
And by the way, you really haven't answered the OP's question. Since you're the freakin' expert, what is the oldest confirmed named person whose body or mummy has actually been located? ←Baseball Bugs What's up, Doc? carrots04:20, 23 March 2011 (UTC)[reply]
I had a feeling there was an even more recent discussion on this, and here it is: Wikipedia:Reference_desk/Archives/Humanities/2010_June_13#First_Notable_Person. It seems Sumerian kings are good candidates. Jørgen (talk) 20:25, 22 March 2011 (UTC)[reply]
Alulim is recorded as ruling somewhere around 275,000BC, though chances are that date is entirely wrong, and even if he ruled only in 4700BC or whenever, there is no direct evidence he certainly existed. Archaeologists have tentatively identified certain graves in Egypt with the earliest known kings there, c3200BC, based on ancient inscriptions amongst the things buried with them. From there on, there is just no one point where it becomes clear that one person was exactly who they think he was, whilst the previous was possibly not. Then again, there is a theoretical possibility the entire universe was created at some impossible to determine point in the recent past, in such a way that it only seemed to have been around much longer. 148.197.121.205 (talk) 10:24, 23 March 2011 (UTC)[reply]

Supply of lawyers deliberately restricted

Lawyers are very expensive, but only a minority of people who hope to have careers as lawyers succeed. Particularly regarding barristers in the UK.

Is there any evidence that the supply of lawyers has been deliberately restricted by the profession so that their fees are kept high? The normal economic rules of supply and demand do not appear to be working. Thanks 92.15.23.133 (talk) 18:09, 22 March 2011 (UTC)[reply]

Yes definitely. I can't speak directly to the situation in the U.K., but in the U.S. there's tremendous barriers to entry that keep the profession smaller than it otherwise would be. Prohibitions against unlicensed practice of law, law school cost and requirement, bar exams, bar fees, and conflict of interest ethical rules all work to increase the demand/reduce supply of legal services. I'm not saying those things aren't necessary to some degree, but they do increase prices.
If you're interested, the ethical rules about conflicts of interest as barriers to entry are the subject of some scholarly debate.
On the flip side, it's expensive to be a lawyer (also for those things). Law school, malpractice insurance, time spent dealing with non-billable matters, long hours, lack of flexibility particularly for those in litigation practices... it's not a free lunch for anybody unfortunately. Shadowjams (talk) 19:53, 22 March 2011 (UTC)[reply]
US lawyers I have discussed this with say there is a vast oversupply of lawyers rather than a "deliberate shortage" of lawyers. Law firms have no work to support the flood of applicants, and a new lawyer is likely to starve if he just opens an office as a sole practitioner, even if his prices are way under those of established firms. One thing that helps the new lawyer in a storefront office is that the old need for shelves full of all sorts of lawbooks is greatly decreased by the ability to do legal research online from a PC (even though the subscription costs are high for the databases). Law schools keep cranking out fresh crops of lawyers, with many graduates unable to find employment practicing law. Here are refs on the US oversupply of lawyers:[17], [18], [19]. As for the UK, here is a news item discussing the oversupply there, where graduates are likely to have to work as paralegals: [20]. All in all. new graduates in the US or the UK will have a hard time paying off the loans most took out to pay for their legal schooling. A very fortunate few got scholarships to pay for law school, or had rich parents who just wrote checks. Edison (talk) 20:35, 22 March 2011 (UTC)[reply]

Checking Account

Is there any reason why a person can not get a checking account (i.e. bad credit)?--Christie the puppy lover (talk) 19:19, 22 March 2011 (UTC)[reply]

In the US, bad credit is the most common reason for banks denying someone a checking account; they think that a past history of poor money decisions makes it more likely the applicant will overdraw their account. A US bank or credit union would also turn down an application in which the person didn't have a government-issued photo ID or Social Security Number. (Here's a short article about the requirements to open an account at most places.) The services of credit unions, which are nonprofits, are cheaper than banks', and their requirements are possibly looser. PS: The applicant needs to be 18 years old or more in the US. Some banks have special "student" checking accounts for people aged 16 and up, but these accounts all need an adult co-signer who promises to be financially responsible in cases of overdrafts. Comet Tuttle (talk) 19:35, 22 March 2011 (UTC)[reply]
To add some detail to what Comet said... a primary reporter for depository institution is Chex Systems, which works much like the credit bureaus but with depository accounts. And yes, they do deny people accounts, although that's a bank-by-bank decision. Shadowjams (talk) 19:48, 22 March 2011 (UTC)[reply]
In the UK, there are current accounts for children that don't allow you to become overdrawn (they simply won't let you withdraw more than you have), and often come with a debit card. You generally need a parent or guardian to cosign, but there's no question of them becoming responsible for your overdraft, because you can't get one. Is there no equivalent in the US? 212.183.128.73 (talk) 10:32, 23 March 2011 (UTC)[reply]
OT: In Australia, fewer and fewer people have cheque (= checking) accounts because the numbers of businesses that accept cheques for payment have dwindled to the point that they're very much the exception rather than the rule now. It's so much easier to swipe a card; and even if that overdraws a credit account, that's a matter between you and your bank, not you and the vendor. -- Jack of Oz [your turn] 20:14, 22 March 2011 (UTC) [reply]
Same here in USA. The only thing I ever use my checkbook for is a particular monthly bill that has to be paid the old-fashioned way. Many people don't bother. APL (talk) 21:36, 22 March 2011 (UTC)[reply]
(ec) Isn't a Checking Account what is known in the UK as a Current Account (typically the account into which your wages or benefits are paid and from which ATM withdrawals and Direct Debits are taken)? It probably doesn't matter whether you actually use a cheque/check book. --Frumpo (talk) 09:15, 23 March 2011 (UTC)[reply]
Added missing "close small" code after Frumpo's sig to prevent effect on following posts. {The poster formerly known as 87.81.230.195} 90.201.110.155 (talk) 11:14, 23 March 2011 (UTC)[reply]

what happened to Luise of Brunswick-Wolfenbuttel 13 other brothers & sisters? — Preceding unsigned comment added by Thomas Routson (talkcontribs) 23:00, 22 March 2011 (UTC)[reply]

Both articles on her parents Antoinette and Ferdinand Albert list her (twelve) siblings, six of which have their own article: Charles I, Elisabeth Christine, Louis Ernest, Ferdinand, Sophie Antoinette, and Juliana Maria. ---Sluzzelin talk 23:35, 22 March 2011 (UTC)[reply]

political violence in middle east vs. geopolitics of oil

How does political violence in Middle East relate to the geopolitics of oil? Is there website where I can read how political violence in Middle East is related to the geopolitics of oil? Thanks. —Preceding unsigned comment added by 70.53.229.103 (talk) 00:04, 23 March 2011 (UTC)[reply]

Resource curse may be relevant. Ariel. (talk) 02:46, 23 March 2011 (UTC)[reply]
Here is a heavily biased and propagandistic article (by the left-wing Center for Research on Globalization) related to political violence and oil, but still worth looking at to know what different people think about the geopolitics of oil. Frankly speaking, most of the claims that political violence and oil are interrelated are inaccurate and pseudoscientific views. --Reference Desker (talk) 04:01, 23 March 2011 (UTC)[reply]
You may well be right, Reference Desker, but since equally you may not be, such a sweeping claim could use some corroboration. {The poster formerly known as 87.81.230 195} 90.201.110.155 (talk) 11:17, 23 March 2011 (UTC)[reply]

factors of Chechnya conflict

What are the central factors of the conflict of Chechnya? What are the causes of it? Is there a website where I can read about it? Thanks. —Preceding unsigned comment added by 70.53.229.103 (talk) 00:06, 23 March 2011 (UTC)[reply]

Well, you can read Chechen War, Chechnya, and so on, or if you'd like a more poetic account, s:The Captive in the Caucasus, etc. Wnt (talk) 00:54, 23 March 2011 (UTC)[reply]
More specifically Origins of the war in Chechnya, Historical basis of the Second Chechen War and Prelude to the Second Chechen War. --Reference Desker (talk) 04:09, 23 March 2011 (UTC)[reply]
And this. --Reference Desker (talk) 04:12, 23 March 2011 (UTC)[reply]
Resentment by many Muslim Caucasus nationalities towards Russian rule goes back to the 19th-century Czarist wars (see Imam Shamil etc.). In the post-Soviet period, the Chechens really have not always been wise in the measures they have chosen to express or further their autonomist or independentist grievances or aspirations -- in the early 1990's, they seemed to take no care as to whether they were provoking a national government which (though less powerful than formerly) still had far more military might than they did, and was under a leadership that was determined to halt the decline and stave off any further territorial fragmentation; while by the end of the 1990s, they were fully embracing the international Wahhabi and/or Taliban and/or al-Qaeda jihad, and launched the aggressive Invasion of Dagestan for the purpose of bringing it under Islamic extremist rule... AnonMoos (talk) 06:39, 23 March 2011 (UTC)[reply]

Profitability of early whaling? (Nantucket, sailing era)

I'm working my way through Moby Dick and have begun to wonder about the profitability of early whaling. According to Melville, ships hunting sperm whales were often gone for 2 to 3 years at a stretch, sometimes even four. Assuming a successful voyage with a full cargo of oil and spermaceti, what kind of profit would a ship like that make back in those times? Was this a lucrative business for all involved? for just the ship financiers? The Masked Booby (talk) 00:49, 23 March 2011 (UTC)[reply]

In the days before kerosene, whale oil was the only lighting oil acceptable in a great number of upper-class and middle-class households; whalebone was the greatly preferred "boning" for women's corsets, etc... AnonMoos (talk) 04:06, 23 March 2011 (UTC)[reply]
Whale oil stood between beeswax candles (for the rich) and beef tallow candles (for the poor) in the early 19th century, and the advent of kerosene or "coal oil" in the mid 19th century. Whale oil was highly desired. Considerations were the cost per unit of illumination, and the amount of soot produced. In the later 19th century, gaslight superseded various oil lights, and in turn was superseded by electric light. Edison (talk) 05:00, 23 March 2011 (UTC)[reply]
Isolating on one particular ship, it would certainly not seem very efficient. But is it reasonable to assume that overall there were whaling ships frequently heading to sea while other whaling ships were coming back into port with their cargo? ←Baseball Bugs What's up, Doc? carrots05:06, 23 March 2011 (UTC)[reply]
There's a whole chapter about profit in this book about American whaling 1816-1906. It's a bit too much for me to make sense of just skimming (and surprisingly complicated—trying to determine true profits in a meaningful way). There's a sort of conclusion on page 457. Apparently profits were "persistently high" from 1817 to the late 1830s, after which "the market moved toward equilibrium" and "a more fundamental downward adjustment...in the late 1850s", then an uptick during the Civil War, followed by a "contraction" of the industry, but "profit rates held up". And finally, "Overall, profit rates seem to have been somewhat higher than returns in other comparable industries..." Of course all of this is about profits for the investors, not the crews. There's another chapter called "Labor" about crew wages. Again, it's complicated. Try starting around page 175 if you want to see how complicated. The book compares wages of various crew positions between whaling and the merchant marine. Apparently whaling officers earned on average "roughly twice as much as those on merchantmen", and captains about three times as much. Wages for ordinary seamen seem to have been lower in whaling than in the merchant service—about a third or a quarter less on average. Then again, the comparison might not be fair. Whaling crews were much more "ethnically diverse", with people of many nationalities, especially Polynesians. Many of these people would not have been able to get work in the US merchant marine. There would be differences in the reason why one would join a whaling crew in the first place, and different expectations and desired outcomes of a voyage. The chapter goes on to explore these kind of issues in great detail. ...Anyway, this book is dense with information on this topic. Very dense! Seems very well researched though. Pfly (talk) 06:37, 23 March 2011 (UTC)[reply]

The Privileges and Immunities Clause of Article Four of the United States Constitution states that "The Citizens of each State shall be entitled to all Privileges and Immunities of Citizens in the several States."

The Texas state government, along with many other U.S. states, operates a public post-secondary education system, in Texas' case the University of Texas System. Admission to University of Texas schools for United States citizens who are not residents of Texas is very difficult; I believe I have read elsewhere that only 10% of students at UT Austin are not Texas residents, and that is a deliberate decision on the part of Texas' legislature.

To me, post-secondary education is a "privilege" given to a citizen of the state of Texas, provided that the citizen can meet the normal entry requirements. Why is it that public universities can discriminate by state of residence in accepting students? NW (Talk) 06:30, 23 March 2011 (UTC)[reply]

There's two closely related P-I clauses: The Privileges and immunities clause and the Privileges or Immunities Clause. If you find the text deviating from the Supreme Court's historic interpretation of it, welcome to Constitutional jurisprudence. I took a brief look at our P&I article and it references the Slaughter House Cases... and while I haven't looked into it beyond that brief glance, I worry that it may be confusing the 14th amendment clause and the article 4 one you're talking about.
Specific to your question, there are two cases you should look at: Supreme Court of New Hampshire v. Piper and Vlandis v. Kline (1973) 412 U.S. 441 (neither of which we apparently have articles for. P&I has a very narrow scope for 2 primary reasons: it only applies to a narrow subset of "fundamental" citizenship rights... and it is also restricted to "citizens", which is a much narrower subset than individuals, and much narrower still than persons. Shadowjams (talk) 06:53, 23 March 2011 (UTC)[reply]
Actually I realize that my answer's pretty bad... I don't think Kline ever addresses Article 4 directly. However I know there's a line of cases on this. I just don't have the resources to find it at the moment. Perhaps someone else can point you to the case I'm thinking of but can't find. Shadowjams (talk) 07:08, 23 March 2011 (UTC)[reply]

help me to get information/contacts for my research based on old woollen trade of Kullu(H.P.)INDIA,especially in British period

Respected sir/mam
i am presently working as a asstt. prof.in government college Banjar distt. Kullu and doing work on the origin of Kullu Handloom shawls and old time woollen trade of Himachal Pradesh with other provincial states especialy during British period.In this topic i find many new findings like how the world famous Kullu shawls came in to being,its connection with Kinnauri Handloom,wich was connected with British trade and interest,opening of new trade roots etc.

Sir,here i want to mention this is the first work ever made on handloom of Kullu shawls and i have compeleted my M.Phil from H P Uiversity Shimla (INDIA).Now i want to do some more hard work but i have no knowledge how to contact any university in England for grant and authentic (contemprary) information/evidence for my reseach.

kindly suggest me. I shall be highly thankful to you for your this act of kindness.

Your's Faithfuly
J.C.Chauhan
e mail add (removed) —Preceding unsigned comment added by 117.239.0.2 (talk) 07:16, 23 March 2011 (UTC)[reply]

Question reformatted for readability, and email removed for privacy AndrewWTaylor (talk) 07:28, 23 March 2011 (UTC)[reply]

British peerage

May I know what is the correct title for Charles Edward Stuart, Count Roehenstart?

Burke's Peerage says "...more commonly known as Charles Edward Stuart, Count de Roehenstart"; The Complete Peerage says "Charles Edward Stuart was styled as Count Roehenstart, self-styled" and then, there is a book entitled, "The pedigree of Charles Edward Stuart, Count of Roehenstart".

Please advise on what the correct way to write his name is. Thanks, Bejinhan talks 10:42, 23 March 2011 (UTC)[reply]